Download as pdf or txt
Download as pdf or txt
You are on page 1of 88

Login ID:8FA03376/Student Name:ANSHULGARG/Overall Score:0

Student ScoreCard
Score:0

Percentile: NA

Test of Logical Reasoning

Score:0

Percentile: NA

Test of Quantitative Aptitude

Score:0

Percentile: NA

Test of VA & RC

Score:0

Percentile: NA

Overall: Overall

Score:0

Percentile: NA

ite
d

Test of Abstract Reasoning

2)

ADBEC

3)

ABCDE

4)

BCADE

Pr

AEBDC

es

1)

iv
at
e

Li
m

Question: 1
Choose from among the alternatives, the option that will form a logical series.

ou

rc

Explanation:
The arrows are numbered from 1 to 4 from top to bottom. The 1starrow moves takes the place of the 4tharrow where as all the remaining
arrows move up by one place. Also all the arrows are laterally inverted with the circle on the arrows in odd numbered places moving to the
other end. Hence, [3].

2)

EACBD

3)

ACDEB

4)

ABCDE

ar

ADBCE

Le

1)

ni

ng

es

Question: 2
Choose from among the alternatives, the option that will form a logical series.

IM

Explanation:
There are three blocks of four elements each. The top left corner elements of each block move anticlockwise among themselves and then,
three blocks move clockwise replacing each other. Also, two bottom elements of the block that move upwards invert laterally and
exchange positions. 'ADBCE' is the correct logical sequence. Hence, [1].
Question: 3
Choose from among the alternatives, the option that will form a logical series.

IMS Learning Resources Pvt.Ltd.,Mumbai.All copyrights to this material vestswith IMS Learning Resources Pvt.Ltd.
No part of this materials either in part oras a whole shall be copied,printed,electronically reproduced,sold or distributed without the written
consent of IMS Learing Resources Pvt.Ltd.and any such violation would entail initiation of suitable legal proceedings.

Copyright

1)

BACDE

2)

ACEBD

3)

AEBDC

4)

BCDEA

ite
d

Login ID:8FA03376/Student Name:ANSHULGARG/Overall Score:0

iv
at
e

Li
m

Explanation:
Of the four elements, the top left element interchanges position with the top right element in each step and inverts vertically in alternate
steps. Also, the top right element is replaced by a new element in alternate steps. Of the two bottom elements, the bottom right element
inverts laterally as well as a element in the bottom left position is replaced by a new element in one step and then in the next step the two
elements interchange positions. The correct logical sequence is 'BCDEA'. Hence, [4].

[2]

3)

[3]

4)

[4]

2)

ng

[1]

ni

1)

es

ou

rc

es

Pr

Question: 4
Choose the option which best continues the series.

Le

ar

Explanation:
The top-left symbol moves to bottom right corner and it rotates 90o clockwise and takes a mirror image. The bottom left symbol moves to
top right corner and it rotates 90oin the anticlockwise direction and takes the water image. The bottom right symbol moves to the bottom
left corner and rotates 90oanticlockwise. The top right symbol moves to the top left corner and rotates by 90oin the clockwise direction.
Hence, [4].

IM

Question: 5
Choose the option which best continues the series.

IMS Learning Resources Pvt.Ltd.,Mumbai.All copyrights to this material vestswith IMS Learning Resources Pvt.Ltd.
No part of this materials either in part oras a whole shall be copied,printed,electronically reproduced,sold or distributed without the written
consent of IMS Learing Resources Pvt.Ltd.and any such violation would entail initiation of suitable legal proceedings.

Copyright

2)

[2]

3)

[3]

4)

[4]

Li
m

[1]

iv
at
e

1)

ite
d

Login ID:8FA03376/Student Name:ANSHULGARG/Overall Score:0

Pr

Explanation:
In the given series, in the first step, the first row and second column elements are kept fixed and the remaining four elements move in the
anticlockwise direction among themselves. In the second step, the second row and third column elements are fixed while in the third step,
the third row and first column elements are kept fixed and in both the steps, the remaining four elements move anticlockwise among
themselves. These steps continue to form the series. Hence, [3].

2)

[2]

3)

[3]
[4]

IM

4)

Le

[1]

1)

ar

ni

ng

es

ou

rc

es

Question: 6
The figure [X] represents a transparent square sheet with a pattern. Choose the option that appears when the sheet is folded along the
dotted line.

Explanation:
Hence, [4].

Question: 7
The figure [X] represents a transparent square sheet with a pattern. Choose the option that appears when the sheet is folded along the
dotted line.

IMS Learning Resources Pvt.Ltd.,Mumbai.All copyrights to this material vestswith IMS Learning Resources Pvt.Ltd.
No part of this materials either in part oras a whole shall be copied,printed,electronically reproduced,sold or distributed without the written
consent of IMS Learing Resources Pvt.Ltd.and any such violation would entail initiation of suitable legal proceedings.

Copyright

[1]

2)

[2]

3)

[3]

4)

[4]

iv
at
e

1)

Li
m

ite
d

Login ID:8FA03376/Student Name:ANSHULGARG/Overall Score:0

Pr

Explanation:
Hence, [3].

[2]

3)

[3]

4)

[4]

IM

Explanation:
Hence, [2].

ar

2)

Le

[1]

1)

ni

ng

es

ou

rc

es

Question: 8
The figure [X] represents a transparent square sheet with a pattern. Choose the option that appears when the sheet is folded along the
dotted line.

Question: 9
The figure [X] represents a transparent square sheet with a pattern. Choose the option that appears when the sheet is folded along the
dotted line.

IMS Learning Resources Pvt.Ltd.,Mumbai.All copyrights to this material vestswith IMS Learning Resources Pvt.Ltd.
No part of this materials either in part oras a whole shall be copied,printed,electronically reproduced,sold or distributed without the written
consent of IMS Learing Resources Pvt.Ltd.and any such violation would entail initiation of suitable legal proceedings.

Copyright

[1]

2)

[2]

3)

[3]

4)

[4]

iv
at
e

1)

Li
m

ite
d

Login ID:8FA03376/Student Name:ANSHULGARG/Overall Score:0

Pr

Explanation:
Hence, [4].

[1]

2)

[2]

3)

[3]

4)

Le

1)

ar

ni

ng

es

ou

rc

es

Question: 10
The figure [X] represents a transparent square sheet with a pattern. Choose the option that appears when the sheet is folded along the
dotted line.

IM

[4]

Explanation:
Hence, [4].

Question: 11
Choose the option which best continues the series.

IMS Learning Resources Pvt.Ltd.,Mumbai.All copyrights to this material vestswith IMS Learning Resources Pvt.Ltd.
No part of this materials either in part oras a whole shall be copied,printed,electronically reproduced,sold or distributed without the written
consent of IMS Learing Resources Pvt.Ltd.and any such violation would entail initiation of suitable legal proceedings.

Copyright

2)

[2]

3)

[3]

4)

[4]

Li
m

[1]

iv
at
e

1)

ite
d

Login ID:8FA03376/Student Name:ANSHULGARG/Overall Score:0

Pr

Explanation:
In each of the figure, the shaded shape disappears and a new shapeis introduced to the place adjacent to the unshaded shape. The
unshaded shape in thefirst figure becomes shaded in the subsequent figure. Therefore, choice 2 comes next inthe series. Hence, [2].

2)

[2]

3)

[3]

4)

[4]

ni

[1]

ar

1)

ng

es

ou

rc

es

Question: 12
Choose the option which best continues the series.

IM

Le

Explanation:
In the given series, unshaded circle and triangle moves inanticlockwise direction. Shaded circle and shaded square moves in clockwise
direction.Square moves in two steps. Due to overlap with shaded elements, some elementsdisappear. This is followed in option [4].
Therefore, choice 4 comes next in the series.Hence, [4].
Question: 13
Choose the option which best continues the series.

IMS Learning Resources Pvt.Ltd.,Mumbai.All copyrights to this material vestswith IMS Learning Resources Pvt.Ltd.
No part of this materials either in part oras a whole shall be copied,printed,electronically reproduced,sold or distributed without the written
consent of IMS Learing Resources Pvt.Ltd.and any such violation would entail initiation of suitable legal proceedings.

Copyright

[1]

2)

[2]

3)

[3]

4)

[4]

iv
at
e

1)

Li
m

ite
d

Login ID:8FA03376/Student Name:ANSHULGARG/Overall Score:0

Pr

Explanation:
Every set of small and large lines rotates 90 in the anticlockwise direction. Also, in every frame a new set appears in the clockwise
position which is an image of the preceding set. Hence, [3].

[2]

3)

[3]

4)

[4]

ar

2)

Le

[1]

1)

ni

ng

es

ou

rc

es

Question: 14
Choose the option which best continues the series.

IM

Explanation:
The number of dots on the front face of the dice is 5, 6, 1, 2, 3 hence there should be 4 dots in the next frame. Also the top face has 2, 3,
4, 5, 6 dots, thus the next frame should have 1 dot. Hence, [3].
Question: 15
Choose the option which best continues the series.

IMS Learning Resources Pvt.Ltd.,Mumbai.All copyrights to this material vestswith IMS Learning Resources Pvt.Ltd.
No part of this materials either in part oras a whole shall be copied,printed,electronically reproduced,sold or distributed without the written
consent of IMS Learing Resources Pvt.Ltd.and any such violation would entail initiation of suitable legal proceedings.

Copyright

2)

[2]

3)

[3]

4)

[4]

Li
m

[1]

iv
at
e

1)

ite
d

Login ID:8FA03376/Student Name:ANSHULGARG/Overall Score:0

Pr

Explanation:
Alternate figures form a series. In the series formed by the first, third and fifth figures, the figure inverts laterally and vertically in alternate
steps. In the series formed by the second and fourth figures, the figure rotates 90 anticlockwise and then the shading shifts to the other
part of the rectangle. Hence, [1].

ng

es

ou

rc

es

Question: 16
Refer to the data below and answer the questions that follow.

ni

123456

ar

1)
2)

Le

3)
4)

IM

Explanation:
The sum of the first and last digit is odd, product of second and fifth is even, sum of third and fourth is odd. Applying all these conditions,
option [3] is code for 123456. Hence, [3].
Question: 17
Refer to the data below and answer the questions that follow.
341068
1)

IMS Learning Resources Pvt.Ltd.,Mumbai.All copyrights to this material vestswith IMS Learning Resources Pvt.Ltd.
No part of this materials either in part oras a whole shall be copied,printed,electronically reproduced,sold or distributed without the written
consent of IMS Learing Resources Pvt.Ltd.and any such violation would entail initiation of suitable legal proceedings.

Copyright

Login ID:8FA03376/Student Name:ANSHULGARG/Overall Score:0

2)
3)
4)
Explanation:
Hence, [1].

ite
d

Question: 18
Refer to the data below and answer the questions that follow.
998877

Li
m

1)
2)
3)

iv
at
e

4)
Explanation:
Hence, [4].

Pr

Question: 19
Refer to the data below and answer the questions that follow.

es

444444

rc

1)
2)

ou

3)

es

4)

Explanation:
Hence, [1].

ng

Question: 20
Refer to the data below and answer the questions that follow.

ni

957123

ar

1)

Le

2)
3)

IM

Explanation:
Hence, [1].

4)

Question: 21
Refer to the data below and answer the questions that follow.

IMS Learning Resources Pvt.Ltd.,Mumbai.All copyrights to this material vestswith IMS Learning Resources Pvt.Ltd.
No part of this materials either in part oras a whole shall be copied,printed,electronically reproduced,sold or distributed without the written
consent of IMS Learing Resources Pvt.Ltd.and any such violation would entail initiation of suitable legal proceedings.

Copyright

1)

2)

3)

4)

1 or 2

iv
at
e

Li
m

ite
d

Login ID:8FA03376/Student Name:ANSHULGARG/Overall Score:0

ar

ni

ng

es

ou

rc

es

Pr

Explanation:

Le

Question: 22
Refer to the data below and answer the questions that follow.

IM

2)

1)

3)

4)

1 or 3

Explanation:

IMS Learning Resources Pvt.Ltd.,Mumbai.All copyrights to this material vestswith IMS Learning Resources Pvt.Ltd.
No part of this materials either in part oras a whole shall be copied,printed,electronically reproduced,sold or distributed without the written
consent of IMS Learing Resources Pvt.Ltd.and any such violation would entail initiation of suitable legal proceedings.

Copyright

Li
m

ite
d

Login ID:8FA03376/Student Name:ANSHULGARG/Overall Score:0

iv
at
e

Question: 23
Refer to the data below and answer the questions that follow.

Pr

1)

es

2)

ou

rc

3)

es

4)

ng

Explanation:

2)

3)

3 or 2

IM

4)

Le

1)

ar

ni

Question: 24
Refer to the data below and answer the questions that follow.

Explanation:

IMS Learning Resources Pvt.Ltd.,Mumbai.All copyrights to this material vestswith IMS Learning Resources Pvt.Ltd.
No part of this materials either in part oras a whole shall be copied,printed,electronically reproduced,sold or distributed without the written
consent of IMS Learing Resources Pvt.Ltd.and any such violation would entail initiation of suitable legal proceedings.

Copyright

Login ID:8FA03376/Student Name:ANSHULGARG/Overall Score:0

ite
d

Question: 25
Refer to the data below and answer the questions that follow.

Li
m

1)

3)

iv
at
e

2)
3

4)

ou

rc

es

Pr

Explanation:

3)

4)

IM

Explanation:

ni

2)

ar

Le

1)

ng

es

Question: 26
Each question has a set of four statements. Each statement has three segments. Choose the alternative where the third segment in the
statement can be logically deduced using both the preceding two but not just one of them.

Question: 27

IMS Learning Resources Pvt.Ltd.,Mumbai.All copyrights to this material vestswith IMS Learning Resources Pvt.Ltd.
No part of this materials either in part oras a whole shall be copied,printed,electronically reproduced,sold or distributed without the written
consent of IMS Learing Resources Pvt.Ltd.and any such violation would entail initiation of suitable legal proceedings.

Copyright

Login ID:8FA03376/Student Name:ANSHULGARG/Overall Score:0

1)

2)

3)

4)

ite
d

Each question has a set of four statements. Each statement has three segments. Choose the alternative where the third segment in the
statement can be logically deduced using both the preceding two but not just one of them.

Pr

iv
at
e

Li
m

Explanation:

2)

B and C

3)

B and D

4)

A and C

es

A and B

1)

ou

rc

es

Question: 28
Each question has a set of four statements. Each statement has three segments. Choose the alternative where the third segment in the
statement can be logically deduced using both the preceding two but not just one of them.

Le

ar

ni

ng

Explanation:

IM

Question: 29
Each question has a set of four statements. Each statement has three segments. Choose the alternative where the third segment in the
statement can be logically deduced using both the preceding two but not just one of them.

1)

IMS Learning Resources Pvt.Ltd.,Mumbai.All copyrights to this material vestswith IMS Learning Resources Pvt.Ltd.
No part of this materials either in part oras a whole shall be copied,printed,electronically reproduced,sold or distributed without the written
consent of IMS Learing Resources Pvt.Ltd.and any such violation would entail initiation of suitable legal proceedings.

Copyright

Login ID:8FA03376/Student Name:ANSHULGARG/Overall Score:0

2)

3)

4)

ite
d

Explanation:

2)

3)

B and C

4)

C and D

Pr

A and B

es

1)

iv
at
e

Li
m

Question: 30
Each question has a set of four statements. Each statement has three segments. Choose the alternative where the third segment in the
statement can be logically deduced using both the preceding two but not just one of them.

ar

Question: 31
Choose the correct alternative.

ni

ng

es

ou

rc

Explanation:

Le

Karlekar is the only brother of Bob who has only one son, Sam whose only aunt is Fitzgerald. How is Karlekar related to Fitzgerald?
Father

2)

Uncle

3)

1)

4)

Cannot be determined

IM

Brother

Explanation:
We know that Karlekar is Bob's only brother, but we don't know how many sisters Bob has. Hence, Fitzgerald can be Karlekar's sister or
his wife. Hence, [4].
Question: 32
Choose the correct alternative.

IMS Learning Resources Pvt.Ltd.,Mumbai.All copyrights to this material vestswith IMS Learning Resources Pvt.Ltd.
No part of this materials either in part oras a whole shall be copied,printed,electronically reproduced,sold or distributed without the written
consent of IMS Learing Resources Pvt.Ltd.and any such violation would entail initiation of suitable legal proceedings.

Copyright

Login ID:8FA03376/Student Name:ANSHULGARG/Overall Score:0

1)

2)

3)

4)

Li
m

ite
d

Explanation:

3)

4)

es

2)

ng

1)

ou

rc

es

Pr

iv
at
e

Question: 33
Choose the correct alternative.

IM

Le

ar

ni

Explanation:

Question: 34
Refer to the data below and answer the questions that follow.

IMS Learning Resources Pvt.Ltd.,Mumbai.All copyrights to this material vestswith IMS Learning Resources Pvt.Ltd.
No part of this materials either in part oras a whole shall be copied,printed,electronically reproduced,sold or distributed without the written
consent of IMS Learing Resources Pvt.Ltd.and any such violation would entail initiation of suitable legal proceedings.

Copyright

ite
d

Login ID:8FA03376/Student Name:ANSHULGARG/Overall Score:0

Lions, dogs, bears

2)

Horses, cats, bears

3)

Bears, horses, dogs

4)

Bears, cats, lions

iv
at
e

1)

Li
m

Which three cannot perform together?

es

ou

rc

es

Pr

Explanation:

ng

Hence, [4].

ni

Question: 35
Refer to the data below and answer the questions that follow.

Bears only

2)

Only bears and lions

3)

Horses, cats, lions

4)

Le

1)

ar

Which are the animal shows that have fixed timings?

IM

Horses, bears, lions

Explanation:

IMS Learning Resources Pvt.Ltd.,Mumbai.All copyrights to this material vestswith IMS Learning Resources Pvt.Ltd.
No part of this materials either in part oras a whole shall be copied,printed,electronically reproduced,sold or distributed without the written
consent of IMS Learing Resources Pvt.Ltd.and any such violation would entail initiation of suitable legal proceedings.

Copyright

Li
m

ite
d

Login ID:8FA03376/Student Name:ANSHULGARG/Overall Score:0

Hence, [1].

[2]

3)

[3]

4)

[4]

IM

Explanation:

ni

2)

ar

[1]

Le

1)

ng

es

ou

rc

es

Pr

iv
at
e

Question: 36
Refer to the data below and answer the questions that follow.

Question: 37
Refer to the data below and answer the questions that follow.

IMS Learning Resources Pvt.Ltd.,Mumbai.All copyrights to this material vestswith IMS Learning Resources Pvt.Ltd.
No part of this materials either in part oras a whole shall be copied,printed,electronically reproduced,sold or distributed without the written
consent of IMS Learing Resources Pvt.Ltd.and any such violation would entail initiation of suitable legal proceedings.

Copyright

Login ID:8FA03376/Student Name:ANSHULGARG/Overall Score:0

1)

[1]

2)

[2]

3)

[3]

4)

[4]

iv
at
e

Li
m

ite
d

Explanation:

[2]

3)

[3]

4)

[4]

es

2)

rc

[1]

ou

1)

Pr

Question: 38
Refer to the data below and answer the questions that follow.

ar

ni

ng

es

Explanation:

[1]

IM

1)

Le

Question: 39
Refer to the data below and answer the questions that follow.

2)

[2]

3)

[3]

4)

[4]

Explanation:

IMS Learning Resources Pvt.Ltd.,Mumbai.All copyrights to this material vestswith IMS Learning Resources Pvt.Ltd.
No part of this materials either in part oras a whole shall be copied,printed,electronically reproduced,sold or distributed without the written
consent of IMS Learing Resources Pvt.Ltd.and any such violation would entail initiation of suitable legal proceedings.

Copyright

Login ID:8FA03376/Student Name:ANSHULGARG/Overall Score:0

2)

[2]

3)

[3]

4)

[4]

Li
m

[1]

iv
at
e

1)

ite
d

Question: 40
Refer to the data below and answer the questions that follow.

ou

rc

es

Pr

Explanation:

2)

South-West

3)

South-East

4)

West

IM

Le

Explanation:

ni

South

ar

1)

ng

es

Question: 41
Choose the correct alternative.

Question: 42

IMS Learning Resources Pvt.Ltd.,Mumbai.All copyrights to this material vestswith IMS Learning Resources Pvt.Ltd.
No part of this materials either in part oras a whole shall be copied,printed,electronically reproduced,sold or distributed without the written
consent of IMS Learing Resources Pvt.Ltd.and any such violation would entail initiation of suitable legal proceedings.

Copyright

Login ID:8FA03376/Student Name:ANSHULGARG/Overall Score:0

Choose the correct alternative.


Alice follows a rabbit as it goes 6 km in the north direction, then 7 km in the west, then goes through an unbent pipe and after coming out
of the pipe takes a right turn and again runs. If Alice is still behind the rabbit and can see it, which direction is she facing now? Given that
the pipe opens in the south-west direction.
North-West

2)

South-West

3)

North-East

4)

South-East

ite
d

1)

iv
at
e

Li
m

Explanation:

es

Pr

Question: 43
Choose the correct alternative.
There is a ground in the shape of a rhombus ABCD, such that the sides of the ground are 5 km each. If a man moves 4 km on diagonal
AC and then turns to his right to walk 2 km, in what direction is he walking, if the ground is such that point C is in the north direction, point
A in the south direction?
East

2)

North

3)

West

4)

Cannot be determined

es

ou

rc

1)

IM

Le

ar

ni

ng

Explanation:

Question: 44
Choose the correct alternative.

IMS Learning Resources Pvt.Ltd.,Mumbai.All copyrights to this material vestswith IMS Learning Resources Pvt.Ltd.
No part of this materials either in part oras a whole shall be copied,printed,electronically reproduced,sold or distributed without the written
consent of IMS Learing Resources Pvt.Ltd.and any such violation would entail initiation of suitable legal proceedings.

Copyright

1)

A's wife

2)

E's wife

3)

A's or E's wife, but not both

4)

I and III only

iv
at
e

Li
m

ite
d

Login ID:8FA03376/Student Name:ANSHULGARG/Overall Score:0

IM

Le

ar

Question: 45
Choose the correct alternative.

ni

ng

es

ou

rc

es

Pr

Explanation:

IMS Learning Resources Pvt.Ltd.,Mumbai.All copyrights to this material vestswith IMS Learning Resources Pvt.Ltd.
No part of this materials either in part oras a whole shall be copied,printed,electronically reproduced,sold or distributed without the written
consent of IMS Learing Resources Pvt.Ltd.and any such violation would entail initiation of suitable legal proceedings.

Copyright

2)

Mike, Ethan

3)

Hunt, Hunt

4)

Ethan, Mike

Li
m

Hunt, Mike

iv
at
e

1)

ite
d

Login ID:8FA03376/Student Name:ANSHULGARG/Overall Score:0

es

ou

rc

es

Pr

Explanation:

2)

ECABD

3)

DABCE

4)

EDCAB

ar

ECADB

Le

1)

ni

ng

Question: 46
Arrange the words/phrases to form a meaningful sentence.

IM

Explanation:
There is a comparative degree in the jumbled sequence i.e., students earn more than the professors who taught them. So we have the
ECA link. Option [2] does not end logically. Hence, [1].
Question: 47
Arrange the words/phrases to form a meaningful sentence.

IMS Learning Resources Pvt.Ltd.,Mumbai.All copyrights to this material vestswith IMS Learning Resources Pvt.Ltd.
No part of this materials either in part oras a whole shall be copied,printed,electronically reproduced,sold or distributed without the written
consent of IMS Learing Resources Pvt.Ltd.and any such violation would entail initiation of suitable legal proceedings.

Copyright

1)

DEABC

2)

BAECD

3)

DABCE

4)

BECDA

ite
d

Login ID:8FA03376/Student Name:ANSHULGARG/Overall Score:0

Li
m

Explanation:
The BE link is the most discernible one. The verb 'linked' in B should be followed by 'to' in E. The BE link is present only in option [4]. If we
follow the sequence accordingly, we will find a meaningful sentence. Hence, [4].

2)

CDAB

3)

BDCA

4)

BDAC

CABD

ng

1)

es

ou

rc

es

Pr

iv
at
e

Question: 48
The sentences given in each question, when properly sequenced, form a coherent paragraph. Each sentence is labelled with a letter.
Choose the most logical order of sentences from among the given choices to construct a coherent paragraph.

ar

ni

Explanation:
B begins the paragraph by introducing the subject of characterisation of living systems. This points to options [3] and [4]. The CA link
cannot be ignored on account of 'these two descriptions'. Hence, [3].

IM

Le

Question: 49
The sentences given in each question, when properly sequenced, form a coherent paragraph. Each sentence is labelled with a letter.
Choose the most logical order of sentences from among the given choices to construct a coherent paragraph.

1)

ABDC

IMS Learning Resources Pvt.Ltd.,Mumbai.All copyrights to this material vestswith IMS Learning Resources Pvt.Ltd.
No part of this materials either in part oras a whole shall be copied,printed,electronically reproduced,sold or distributed without the written
consent of IMS Learing Resources Pvt.Ltd.and any such violation would entail initiation of suitable legal proceedings.

Copyright

Login ID:8FA03376/Student Name:ANSHULGARG/Overall Score:0

2)

BDAC

3)

CBDA

4)

DACB

Explanation:
CB are co-ordinately linked (merely, also). A provides the appropriate conclusion. Hence, [3].

2)

CDEBA

3)

CBDAE

4)

CEBDA

es

CBDEA

rc

1)

Pr

iv
at
e

Li
m

ite
d

Question: 50
The sentences given in each question, when properly sequenced, form a coherent paragraph. Each sentence is labelled with a letter.
Choose the most logical order of sentences from among the given choices to construct a coherent paragraph.

es

ou

Explanation:
Starting with C, note that B and not D or E is the next sentence. Choosing between [l] and [3], we see that [3] is logically consistent. (Hint:
The DA link). Hence, [3].

ar

ni

ng

Question: 51
Refer to the data below and answer the questions that follow.

1)

Daughter

2)

Daughter-in-law

3)

Le

If X $ Z > P * Q, then how is Q related to X?

IM

Mother

4)

Granddaughter-in-law

Explanation:

IMS Learning Resources Pvt.Ltd.,Mumbai.All copyrights to this material vestswith IMS Learning Resources Pvt.Ltd.
No part of this materials either in part oras a whole shall be copied,printed,electronically reproduced,sold or distributed without the written
consent of IMS Learing Resources Pvt.Ltd.and any such violation would entail initiation of suitable legal proceedings.

Copyright

Login ID:8FA03376/Student Name:ANSHULGARG/Overall Score:0

Li
m

ite
d

Question: 52
Refer to the data below and answer the questions that follow.

1)

Mother

2)

Mother-in-law

3)

Father

4)

Either Mother-in-law or Father-in-law

Pr

iv
at
e

If P > R $ S ~ Q, then how Q is related to R?

es

ou

rc

es

Explanation:

3)

E/B

4)

ar

2)

Le

1)

ni

ng

Question: 53
Refer to the data below and answer the questions that follow.
Ramesh bought five different greeting cards for five of his friends A, B, C, D and E. He put names of his friends on each of the cards and
on their envelopes. All the envelopes were identical. However, his son took out all the cards from their envelopes and put them all into
wrong envelopes. Unaware of this, Ramesh gave his friends the cards according to the names on the envelopes.
If A got D's card, D got E's card and C got A's card, then who got C's card?

IM

Explanation:
There are five cards and five envelopes having names A, B, C, D andE. None of the cards are into proper envelopes.

Question: 54
Refer to the data below and answer the questions that follow.

IMS Learning Resources Pvt.Ltd.,Mumbai.All copyrights to this material vestswith IMS Learning Resources Pvt.Ltd.
No part of this materials either in part oras a whole shall be copied,printed,electronically reproduced,sold or distributed without the written
consent of IMS Learing Resources Pvt.Ltd.and any such violation would entail initiation of suitable legal proceedings.

Copyright

Login ID:8FA03376/Student Name:ANSHULGARG/Overall Score:0

1)

I, II and III

2)

Only II

3)

II and III

4)

I and II

ite
d

If A and D got each others cards, then which of the following statement/s independentlyis/are necessarily false?
I. E has B's card.
II. B and C have each others card.
III.Either B or C has E's card.
IV. B has C's card and C has E's card.

Pr

iv
at
e

Li
m

Explanation:
There are five cards and five envelopes having names A, B, C, D andE. None of the cards are into proper envelopes.

rc

es

Question: 55
Refer to the data below and answer the questions that follow.
If C and E get B and D's cards in some order, what is the probability that B gets A's card?

ou

1)

3)

es

2)

ng

4)

IM

Le

ar

ni

Explanation:

Question: 56
Each question has three statements followed by five options. Choose the option that follows from all the given statements.

IMS Learning Resources Pvt.Ltd.,Mumbai.All copyrights to this material vestswith IMS Learning Resources Pvt.Ltd.
No part of this materials either in part oras a whole shall be copied,printed,electronically reproduced,sold or distributed without the written
consent of IMS Learing Resources Pvt.Ltd.and any such violation would entail initiation of suitable legal proceedings.

Copyright

Login ID:8FA03376/Student Name:ANSHULGARG/Overall Score:0

I. All pens are pencils.


II. All pencils are books.
III. Some books are notepads.
All books are pens.

2)

All books are pencils.

3)

Some pens are books.

4)

None of these.

ite
d

1)

Pr

iv
at
e

Li
m

Explanation:

Some vans are cars.

2)

All lorries are cars.

3)

Some vans are not cars.

4)

None of these.

IM

Le

ar

ni

ng

Explanation:

1)

es

ou

rc

es

Question: 57
Each question has three statements followed by five options. Choose the option that follows from all the given statements.
I. Some cars are motorbikes.
II. Some motorbikes are vans.
III. No lorries are motorbikes.

IMS Learning Resources Pvt.Ltd.,Mumbai.All copyrights to this material vestswith IMS Learning Resources Pvt.Ltd.
No part of this materials either in part oras a whole shall be copied,printed,electronically reproduced,sold or distributed without the written
consent of IMS Learing Resources Pvt.Ltd.and any such violation would entail initiation of suitable legal proceedings.

Copyright

Pr

iv
at
e

Li
m

ite
d

Login ID:8FA03376/Student Name:ANSHULGARG/Overall Score:0

Some dogs are parrots.

3)

Some dogs are pets.

4)

None of these.

ou

2)

es

Some cats are pets.

1)

rc

es

Question: 58
Each question has three statements followed by five options. Choose the option that follows from all the given statements.
I. All dogs are cats.
II. Some cats are parrots.
III. All parrots are pets.

IM

Le

ar

ni

ng

Explanation:

IMS Learning Resources Pvt.Ltd.,Mumbai.All copyrights to this material vestswith IMS Learning Resources Pvt.Ltd.
No part of this materials either in part oras a whole shall be copied,printed,electronically reproduced,sold or distributed without the written
consent of IMS Learing Resources Pvt.Ltd.and any such violation would entail initiation of suitable legal proceedings.

Copyright

rc

es

Pr

iv
at
e

Li
m

ite
d

Login ID:8FA03376/Student Name:ANSHULGARG/Overall Score:0

Some swords are pistols.

2)

Some pistols are guns.

3)

Some swords are not knives.

4)

None of these.

ar

ni

ng

1)

es

ou

Question: 59
Each question has three statements followed by five options. Choose the option that follows from all the given statements.
I. All swords are knives.
II. All pistols are knives.
III. Some knives are guns.

IM

Le

Explanation:

IMS Learning Resources Pvt.Ltd.,Mumbai.All copyrights to this material vestswith IMS Learning Resources Pvt.Ltd.
No part of this materials either in part oras a whole shall be copied,printed,electronically reproduced,sold or distributed without the written
consent of IMS Learing Resources Pvt.Ltd.and any such violation would entail initiation of suitable legal proceedings.

Copyright

ni

ng

es

ou

rc

es

Pr

iv
at
e

Li
m

ite
d

Login ID:8FA03376/Student Name:ANSHULGARG/Overall Score:0

Le

ar

Question: 60
Each question has three statements followed by five options. Choose the option that follows from all the given statements.

IM

I. All movies are episodes.


II. All episodes are comedies.
III. No comedies are boring.
1)

All movies are comedies.

2)

Some movies are boring.

3)

All comedies are movies.

4)

None of these.

Explanation:

IMS Learning Resources Pvt.Ltd.,Mumbai.All copyrights to this material vestswith IMS Learning Resources Pvt.Ltd.
No part of this materials either in part oras a whole shall be copied,printed,electronically reproduced,sold or distributed without the written
consent of IMS Learing Resources Pvt.Ltd.and any such violation would entail initiation of suitable legal proceedings.

Copyright

Li
m

ite
d

Login ID:8FA03376/Student Name:ANSHULGARG/Overall Score:0

ou

rc

es

Pr

iv
at
e

Question: 61
Refer to the data below and answer the questions that follow.
The sequence of words is fed into a machine. The machine then processes that sequence in certain steps to give an output as shown
below:
Input: she sells sea shells the on sea shore.
Step I: she sells sea shells the on shore sea.
Step II: she sells sea shells the shore on sea.
Step III: she sells sea shells the shore on sea.
Step IV: she sells sea the shells shore on sea.
Step V: she sells the sea shells shore on sea.
Step VI: she the sells sea shells shore on sea.
Step VII: the she sells sea shells shore on sea.
Step VII is the output.

2)

II

3)

VII

4)

VI

III

ng

1)

es

If input is 'betty bought a butter', then what will be the last step?

IM

Le

ar

ni

Explanation:
In the given input, the last word is compared with the second last word and the one which comes first according to the dictionary moves to
the right. Then the 2ndlast word is compared with the 3rd last and the word which comes first in the dictionary is moved to 2ndlast position
and so on.
Input: betty bought a butter
Step I: betty bought butter a
Step II: betty butter bought a
Step III: butter betty bought a
Hence, [1].
Question: 62
Refer to the data below and answer the questions that follow.
If input is 'beggars are choosers not', then what will be the output?
1)

beggars not are choosers

IMS Learning Resources Pvt.Ltd.,Mumbai.All copyrights to this material vestswith IMS Learning Resources Pvt.Ltd.
No part of this materials either in part oras a whole shall be copied,printed,electronically reproduced,sold or distributed without the written
consent of IMS Learing Resources Pvt.Ltd.and any such violation would entail initiation of suitable legal proceedings.

Copyright

Login ID:8FA03376/Student Name:ANSHULGARG/Overall Score:0

2)

beggars are not choosers

3)

not beggars are choosers

4)

not choosers beggars are

Li
m

ite
d

Explanation:
In the given input, the last word is compared with the second lastword and the one which comes first according to the dictionary moves to
the right. Thenthe 2ndlast word is compared with the 3rd last and the word which comes first in thedictionary is moved to 2ndlast position
and so on.Input: beggars are choosers not
Step I: beggars are not choosers
Step II: beggars not are choosers
Step III: not beggars are choosers
Hence, [3].
Question: 63
Refer to the data below and answer the questions that follow.

beggars

2)

are

3)

not

4)

choosers

Pr

1)

iv
at
e

If the input in the above question is fed into the machine, then what will be the third word from left in the third step?

ou

rc

es

Explanation:
In the given input, the last word is compared with the second lastword and the one which comes first according to the dictionary moves to
the right. Thenthe 2ndlast word is compared with the 3rd last and the word which comes first in thedictionary is moved to 2ndlast position
and so on.The 3rdword from left in the 3rdstep is 'are'.Hence, [2].

es

Question: 64
Refer to the data below and answer the questions that follow.

Input: silver cloud every has lining


What will be the string formed in the last step for the above input?
silver lining cloud every has

2)

lining silver has every cloud

3)

silver lining has every cloud

4)

every cloud silver lining

ar

ni

ng

1)

IM

Le

Explanation:
In the given input, the last word is compared with the second last word and the one which comes first according to the dictionary moves to
the right. Then the 2ndlast word is compared with the 3rd last and the word which comes first in the dictionary is moved to 2ndlast position
and so on.
Input: silver cloud every has lining
Step I: silver cloud every lining has
Step II: silver cloud lining every has
Step III: silver lining cloud every has
Step IV: silver lining cloud every has
Hence, [1].
Question: 65

IMS Learning Resources Pvt.Ltd.,Mumbai.All copyrights to this material vestswith IMS Learning Resources Pvt.Ltd.
No part of this materials either in part oras a whole shall be copied,printed,electronically reproduced,sold or distributed without the written
consent of IMS Learing Resources Pvt.Ltd.and any such violation would entail initiation of suitable legal proceedings.

Copyright

Login ID:8FA03376/Student Name:ANSHULGARG/Overall Score:0

Refer to the data below and answer the questions that follow.
Input: barking dogs seldom bite do. What is the second word from left in the third step when the given input is fed into the machine?
1)

barking

2)

dogs

3)

seldom

4)

bite

iv
at
e

Li
m

ite
d

Explanation:
In the given input, the last word is compared with the second last word and the one which comes first according to the dictionary moves to
the right. Then the 2ndlast word is compared with the 3rd last and the word which comes first in the dictionary is moved to 2ndlast position
and so on.
Input: barking dogs seldom bite do
Step I: barking dogs seldom do bite
Step II: barking dogs seldom do bite
Step III: barking seldom dogs do bite
Hence, [3].

1298

3)

1364

4)

1548

es

2)

rc

1369

ou

1)

Pr

Question: 66
Choose the option that will best continue the series.
28, 117, 336, 775, ____.

ni

ng

es

Explanation:

2)

54

3)

85
96

IM

4)

Le

45

1)

ar

Question: 67
Choose the option that will best continue the series.
3, 13, 7, 41, 11, ____.

Explanation:
The series of perfect squares is 1, 4, 9, 16, 25, 36, 49, ...
The 1stterm is the difference of the 1sttwo terms in the series of perfect squares.
The 2ndterm is the sum of the 2ndand the 3rdterms in the series of perfect squares.
The 3rdterm is the difference in the 3rdand the 4thterms in the series of perfect squaresand so on.
Hence, the next term is 36 + 49 = 85.
Hence, [3].

IMS Learning Resources Pvt.Ltd.,Mumbai.All copyrights to this material vestswith IMS Learning Resources Pvt.Ltd.
No part of this materials either in part oras a whole shall be copied,printed,electronically reproduced,sold or distributed without the written
consent of IMS Learing Resources Pvt.Ltd.and any such violation would entail initiation of suitable legal proceedings.

Copyright

Login ID:8FA03376/Student Name:ANSHULGARG/Overall Score:0

Question: 68
Choose the option that will best continue the series.
1, 11, 39, 89, ____.
127

2)

198

3)

167

4)

173

ite
d

1)

iv
at
e

Li
m

Explanation:

Pr

Question: 69
Choose the option that will best continue the series.

2)

1237

3)

7523

4)

1346

rc

1759

ou

1)

es

35, 142, 397, 896, ____.

ni

ng

es

Explanation:

IM

Le

ar

Question: 70
Each question is followed by two statements I and II. Answer each question using the following instructions.
Mark [1]; if the question can be answered by using only one statement alone.
Mark [2]; if the question can be answered by using both the statements together, but cannot be answered by either of the statements
alone.
Mark [3]; if the question can be answered by using either of the statements alone.
Mark [4]; if more data is required to answer the question.

1)

[1]

2)

[2]

3)

[3]

IMS Learning Resources Pvt.Ltd.,Mumbai.All copyrights to this material vestswith IMS Learning Resources Pvt.Ltd.
No part of this materials either in part oras a whole shall be copied,printed,electronically reproduced,sold or distributed without the written
consent of IMS Learing Resources Pvt.Ltd.and any such violation would entail initiation of suitable legal proceedings.

Copyright

Login ID:8FA03376/Student Name:ANSHULGARG/Overall Score:0

4)

[4]

Explanation:
From statement I: Viren > Jude > Yogi > Amit.
Hence, statement I alone is sufficient.
Statement II alone is clearly not sufficient.
Hence, [1].

Only I follows

2)

I and III follow

3)

II and IV follow

4)

None follow

Pr

1)

iv
at
e

Li
m

ite
d

Question: 71
Each question has two statements followed by four conclusions. Mark the option where the conclusion logically follows from the two
statements.

ar

ni

ng

es

ou

rc

es

Explanation:

IM

Le

Question: 72
Each question has two statements followed by four conclusions. Mark the option where the conclusion logically follows from the two
statements.

1)

I and III follow

2)

Only I follows

IMS Learning Resources Pvt.Ltd.,Mumbai.All copyrights to this material vestswith IMS Learning Resources Pvt.Ltd.
No part of this materials either in part oras a whole shall be copied,printed,electronically reproduced,sold or distributed without the written
consent of IMS Learing Resources Pvt.Ltd.and any such violation would entail initiation of suitable legal proceedings.

Copyright

Login ID:8FA03376/Student Name:ANSHULGARG/Overall Score:0

3)

Only II follows

4)

I and IV follow

iv
at
e

Li
m

ite
d

Explanation:

Only IV follows

3)

Only III follows

4)

I and II follow

ou

2)

es

II and IV follow

1)

rc

es

Pr

Question: 73
Each question has two statements followed by four conclusions. Mark the option where the conclusion logically follows from the two
statements.

IM

Le

ar

ni

ng

Explanation:

Question: 74
Each question has two statements followed by four conclusions. Mark the option where the conclusion logically follows from the two
statements.

IMS Learning Resources Pvt.Ltd.,Mumbai.All copyrights to this material vestswith IMS Learning Resources Pvt.Ltd.
No part of this materials either in part oras a whole shall be copied,printed,electronically reproduced,sold or distributed without the written
consent of IMS Learing Resources Pvt.Ltd.and any such violation would entail initiation of suitable legal proceedings.

Copyright

1)

I and II follow

2)

II and III follow

3)

Only II follows

4)

Only IV follows

ite
d

Login ID:8FA03376/Student Name:ANSHULGARG/Overall Score:0

rc

es

Pr

iv
at
e

Li
m

Explanation:

2)

Only I follows

3)

III and IV follow

4)

II and IV follow

ni

Only II follows

Le

ar

1)

ng

es

ou

Question: 75
Each question has two statements followed by four conclusions. Mark the option where the conclusion logically follows from the two
statements.

IM

Explanation:

IMS Learning Resources Pvt.Ltd.,Mumbai.All copyrights to this material vestswith IMS Learning Resources Pvt.Ltd.
No part of this materials either in part oras a whole shall be copied,printed,electronically reproduced,sold or distributed without the written
consent of IMS Learing Resources Pvt.Ltd.and any such violation would entail initiation of suitable legal proceedings.

Copyright

ite
d

Login ID:8FA03376/Student Name:ANSHULGARG/Overall Score:0

ni

ng

es

ou

rc

es

Pr

iv
at
e

Li
m

Question: 76
Refer to the data given below to answer the questions that follow.

[1]

2)

[2]

3)

[3]

4)

Le

1)

ar

Shoaib has been working in the Agmax BPO for past 3 years. He is B.Sc. in Computer Science with 65% from Mumbai University. He
cleared both the Aptitude test and Voice and Accent round. He also cleared the process interview.

IM

[4]

Explanation:
Shoaib satisfies all the conditions. Therefore, he will be selected for the post of CCE. Hence, [1].
Question: 77
Refer to the data given below to answer the questions that follow.
Neha is BA in English from Delhi University with an aggregate of 64%. She cleared both the Aptitude test and Voice and Accent round.
After graduating from the University, she has been working with Callos BPO for past 22 months. She has also cleared her Process

IMS Learning Resources Pvt.Ltd.,Mumbai.All copyrights to this material vestswith IMS Learning Resources Pvt.Ltd.
No part of this materials either in part oras a whole shall be copied,printed,electronically reproduced,sold or distributed without the written
consent of IMS Learing Resources Pvt.Ltd.and any such violation would entail initiation of suitable legal proceedings.

Copyright

Login ID:8FA03376/Student Name:ANSHULGARG/Overall Score:0

Interview.
1)

[1]

2)

[2]

3)

[3]

4)

[4]

ite
d

Explanation:
Neha satisfies all the conditions but she has less than 2 years of experience in the BPO industry. Therefore, she will be referred to the
Process Manager. Hence, [3].

Li
m

Question: 78
Refer to the data given below to answer the questions that follow.

[1]

2)

[2]

3)

[3]

4)

[4]

Pr

1)

iv
at
e

Kavita is working in BPO industry for past 5 years. She cleared the Aptitude test, the Voice and Accent round and the Process Interview.
She is B.E. in Civil Engineering with 70% aggregate marks.

rc

ou

Question: 79
Refer to the data given below to answer the questions that follow.

es

Explanation:
Kavita is from engineering background. Therefore, she will be rejected. Hence, [4].

2)

[2]

3)

[3]

4)

[4]

[1]

ng

1)

es

Gurpreet is a BDS with 75% aggregate marks. He is working in BPO industry for past 25 months. He cleared the Aptitude test, the Voice
and Accent round and the Process Interview.

ar

ni

Explanation:
Gurpreet satisfies all the conditions. Therefore, he will be selected for the post of CCE. Hence, [1].

Le

Question: 80
Refer to the data given below to answer the questions that follow.

showing Hence

IM

Gaurav has done B. Pharma with 61 % aggregate marks . He cleared the process interview but failed in the Aptitude test. He cleared the
Voice and Accent round. He has 4 years of experience in the BPO industry.
1)

[1]

2)

[2]

3)

[3]

4)

[4]

Explanation:
Gaurav satisfies all the conditions but could not clear the Aptitude test. Therefore, he will be sent to special 2 month training. Hence, [2].

IMS Learning Resources Pvt.Ltd.,Mumbai.All copyrights to this material vestswith IMS Learning Resources Pvt.Ltd.
No part of this materials either in part oras a whole shall be copied,printed,electronically reproduced,sold or distributed without the written
consent of IMS Learing Resources Pvt.Ltd.and any such violation would entail initiation of suitable legal proceedings.

Copyright

Login ID:8FA03376/Student Name:ANSHULGARG/Overall Score:0

Question: 81
Refer to the data below and answer the questions that follow.
Each English alphabet is assigned a code, such that A is coded as 1, B is coded as 2, C is coded as 3, D as 4 and so on.

24

2)

38

3)

28

4)

32

Li
m

1)

ite
d

Which of the following will be the code for the word 'BOY', if the code of the word is the sum of the codes of the individual letters in the
word?

iv
at
e

Explanation:

Pr

Question: 82
Refer to the data below and answer the questions that follow.

2)

3324

3)

4446

4)

4446

rc

3324

ou

1)

es

Which one of these will be the code for 'MARS', if the code of the word is the product of the codes of the individual letters in the word?

es

Explanation:

16

3)

4)

12

IM

Explanation:

ar

2)

Le

1)

ni

ng

Question: 83
Refer to the data below and answer the questions that follow.
Which of these will be the code for 'SALT', if the code of the word is the sum of the codes of the individual letters in the word?

Question: 84
Refer to the data below and answer the questions that follow.
4 out of the 5 groups of letters are alike in a certain way. Find the odd one out.
1)

W, S, K, U

IMS Learning Resources Pvt.Ltd.,Mumbai.All copyrights to this material vestswith IMS Learning Resources Pvt.Ltd.
No part of this materials either in part oras a whole shall be copied,printed,electronically reproduced,sold or distributed without the written
consent of IMS Learing Resources Pvt.Ltd.and any such violation would entail initiation of suitable legal proceedings.

Copyright

Login ID:8FA03376/Student Name:ANSHULGARG/Overall Score:0

2)

E, C, G, K

3)

G, M, B, E

4)

M, Q, S, W

Explanation:
The absolute value of the codes of the alphabets belonging to the groups in options [2], [3] and [4] are prime numbers. However in option
[1], one letter U has code 21, which is not a prime number. Hence, [1].

ite
d

Question: 85
Refer to the data below and answer the questions that follow.

80256

2)

80256

3)

71820

4)

76608

iv
at
e

1)

Li
m

Which of these will be the code for 'PLUS', if the code of the word is the product of the codes of the individual letters in the word?

Pr

Explanation:

2)

XPUJM

3)

ULMVZ

4)

XLPJM

IM

Explanation:

ar

ULPJM

Le

1)

ni

What will be the code for 31247?

ng

es

ou

rc

es

Question: 86
Refer to the data below and answer the questions that follow.

Question: 87

IMS Learning Resources Pvt.Ltd.,Mumbai.All copyrights to this material vestswith IMS Learning Resources Pvt.Ltd.
No part of this materials either in part oras a whole shall be copied,printed,electronically reproduced,sold or distributed without the written
consent of IMS Learing Resources Pvt.Ltd.and any such violation would entail initiation of suitable legal proceedings.

Copyright

Login ID:8FA03376/Student Name:ANSHULGARG/Overall Score:0

Refer to the data below and answer the questions that follow.
What will be the code for 21894?
1)

YLVZJ

2)

PVZJY

3)

PLVZY

4)

YLPVM

Li
m

ite
d

Explanation:

2)

XVJUL

3)

MVJLU

4)

MJVLY

Pr

XJVUL

es

1)

iv
at
e

Question: 88
Refer to the data below and answer the questions that follow.
What will be the code for 78431?

es

ou

rc

Explanation:

2)

JZWTY

3)

XZTVY

4)

JZWTM

IM

Explanation:

ar

XZWTL

Le

1)

ni

ng

Question: 89
Refer to the data below and answer the questions that follow.
What will be the code for 49567?

Question: 90
Refer to the data below and answer the questions that follow.
What will be the code for 32852?

IMS Learning Resources Pvt.Ltd.,Mumbai.All copyrights to this material vestswith IMS Learning Resources Pvt.Ltd.
No part of this materials either in part oras a whole shall be copied,printed,electronically reproduced,sold or distributed without the written
consent of IMS Learing Resources Pvt.Ltd.and any such violation would entail initiation of suitable legal proceedings.

Copyright

Login ID:8FA03376/Student Name:ANSHULGARG/Overall Score:0

1)

XPVWY

2)

XJVZY

3)

YJXLY

4)

XPVWX

Li
m

ite
d

Explanation:

IM

Le

ar

ni

ng

es

ou

rc

es

Pr

iv
at
e

Question: 91
Refer to the data below and answer the questions that follow.

In 2002-2003, what percentage of PBT was paid as tax?


1)

10%

2)

29%

3)

45%

4)

54%

Explanation:

IMS Learning Resources Pvt.Ltd.,Mumbai.All copyrights to this material vestswith IMS Learning Resources Pvt.Ltd.
No part of this materials either in part oras a whole shall be copied,printed,electronically reproduced,sold or distributed without the written
consent of IMS Learing Resources Pvt.Ltd.and any such violation would entail initiation of suitable legal proceedings.

Copyright

ite
d

Login ID:8FA03376/Student Name:ANSHULGARG/Overall Score:0

Li
m

Question: 92
Refer to the data below and answer the questions that follow.

1)

1997-1998

2)

2000-2001

3)

2001-2002

4)

2002-2003

iv
at
e

In which year was the operating efficiency the highest?

IM

Le

ar

ni

ng

es

ou

rc

es

Pr

Explanation:

Question: 93
Refer to the data below and answer the questions that follow.

IMS Learning Resources Pvt.Ltd.,Mumbai.All copyrights to this material vestswith IMS Learning Resources Pvt.Ltd.
No part of this materials either in part oras a whole shall be copied,printed,electronically reproduced,sold or distributed without the written
consent of IMS Learing Resources Pvt.Ltd.and any such violation would entail initiation of suitable legal proceedings.

Copyright

Login ID:8FA03376/Student Name:ANSHULGARG/Overall Score:0

What the approximate average sales for the period 1997-1998 to 2003-2004 (in Rs.crore)?
1)

550

2)

500

3)

525

4)

575

ite
d

Explanation:

Li
m

Question: 94
Refer to the data below and answer the questions that follow.

Operating cost

2)

Interest

3)

Depreciation

4)

Tax

Pr

1)

iv
at
e

In 2003-2004, the net profit declined inspite of an increase in sales due to a greater proportion (as a proportion of sales) of which one of
the following?

Le

ar

ni

ng

es

ou

rc

es

Explanation:

IM

Question: 95
Refer to the data below and answer the questions that follow.

1)

I and II

2)

II and III

3)

I and III

4)

Only III

IMS Learning Resources Pvt.Ltd.,Mumbai.All copyrights to this material vestswith IMS Learning Resources Pvt.Ltd.
No part of this materials either in part oras a whole shall be copied,printed,electronically reproduced,sold or distributed without the written
consent of IMS Learing Resources Pvt.Ltd.and any such violation would entail initiation of suitable legal proceedings.

Copyright

Login ID:8FA03376/Student Name:ANSHULGARG/Overall Score:0

ng

es

ou

rc

es

Pr

iv
at
e

Li
m

ite
d

Explanation:

IM

Le

ar

ni

Question: 96
Refer to the data below and answer the questions that follow.

IMS Learning Resources Pvt.Ltd.,Mumbai.All copyrights to this material vestswith IMS Learning Resources Pvt.Ltd.
No part of this materials either in part oras a whole shall be copied,printed,electronically reproduced,sold or distributed without the written
consent of IMS Learing Resources Pvt.Ltd.and any such violation would entail initiation of suitable legal proceedings.

Copyright

2)

1.925

3)

2.025

4)

2.125

es

1.875

rc

1)

Pr

What is the total number (in crores) of network 'Sodafone' subscribers in all the cities?

iv
at
e

Li
m

ite
d

Login ID:8FA03376/Student Name:ANSHULGARG/Overall Score:0

Le

ar

ni

ng

es

ou

Explanation:

Question: 97
Refer to the data below and answer the questions that follow.

IM

In the two cities which have equal number of network Lidea subscribers, what is the ratio of network sodafone subscribers combined to
network hairtel subscribers combined?
1)

13:16

2)

14:15

3)

16:13

4)

15:14

Explanation:

IMS Learning Resources Pvt.Ltd.,Mumbai.All copyrights to this material vestswith IMS Learning Resources Pvt.Ltd.
No part of this materials either in part oras a whole shall be copied,printed,electronically reproduced,sold or distributed without the written
consent of IMS Learing Resources Pvt.Ltd.and any such violation would entail initiation of suitable legal proceedings.

Copyright

Question: 98
Refer to the data below and answer the questions that follow.

40

2)

42

3)

37

4)

45

iv
at
e

1)

Li
m

What is the average number (in lakhs) of mobile users subscribing to anyone of these three networks in Kolkata?

ite
d

Login ID:8FA03376/Student Name:ANSHULGARG/Overall Score:0

es

ou

Question: 99
Refer to the data below and answer the questions that follow.

rc

es

Pr

Explanation:

2)

25%

3)

33.33%

4)

38.33%

ni

20%

ar

1)

ng

In Mumbai, the number of network hairtel and Lidea subscribers combined is what percentage higher than the number of network
sodafone subscribers?

IM

Le

Explanation:

Question: 100
Refer to the data below and answer the questions that follow.
What percentage of the mobile users in New Delhi are network hairtel subscribers?

IMS Learning Resources Pvt.Ltd.,Mumbai.All copyrights to this material vestswith IMS Learning Resources Pvt.Ltd.
No part of this materials either in part oras a whole shall be copied,printed,electronically reproduced,sold or distributed without the written
consent of IMS Learing Resources Pvt.Ltd.and any such violation would entail initiation of suitable legal proceedings.

Copyright

Login ID:8FA03376/Student Name:ANSHULGARG/Overall Score:0

1)

34%

2)

25.93%

3)

28%

4)

38%

Li
m

ite
d

Explanation:

Pr

iv
at
e

Question: 101
Choose the correct alternative.

es

1)

rc

2)

30

es

4)

ou

3)

ar

ni

ng

Explanation:

Le

Question: 102
Choose the correct alternative.

194

IM

2)

193.53

1)

3)

75

4)

89

Explanation:

IMS Learning Resources Pvt.Ltd.,Mumbai.All copyrights to this material vestswith IMS Learning Resources Pvt.Ltd.
No part of this materials either in part oras a whole shall be copied,printed,electronically reproduced,sold or distributed without the written
consent of IMS Learing Resources Pvt.Ltd.and any such violation would entail initiation of suitable legal proceedings.

Copyright

Li
m

ite
d

Login ID:8FA03376/Student Name:ANSHULGARG/Overall Score:0

2)

3)

4)

81

es

27

rc

1)

Pr

iv
at
e

Question: 103
Choose the correct alternative.

IM

Le

ar

ni

ng

es

ou

Explanation:

Question: 104
Choose the correct alternative.

1)

14

IMS Learning Resources Pvt.Ltd.,Mumbai.All copyrights to this material vestswith IMS Learning Resources Pvt.Ltd.
No part of this materials either in part oras a whole shall be copied,printed,electronically reproduced,sold or distributed without the written
consent of IMS Learing Resources Pvt.Ltd.and any such violation would entail initiation of suitable legal proceedings.

Copyright

Login ID:8FA03376/Student Name:ANSHULGARG/Overall Score:0

2)

25

3)

15

4)

196

Li
m

ite
d

Explanation:

2)

3)

4)

Pr

es

1)

iv
at
e

Question: 105
Choose the correct alternative.

Le

ar

Question: 106
Choose the correct alternative.

ni

ng

es

ou

rc

Explanation:

A town has a provision for food for 68 days. If after 8 days 450 more people are added, then the food lasts for only 65 days. How many
people are there in the town?
9450

2)

1)

3)

9000

4)

8550

IM

9500

Explanation:

IMS Learning Resources Pvt.Ltd.,Mumbai.All copyrights to this material vestswith IMS Learning Resources Pvt.Ltd.
No part of this materials either in part oras a whole shall be copied,printed,electronically reproduced,sold or distributed without the written
consent of IMS Learing Resources Pvt.Ltd.and any such violation would entail initiation of suitable legal proceedings.

Copyright

Login ID:8FA03376/Student Name:ANSHULGARG/Overall Score:0

Rs.500000

2)

Rs.450000

3)

Rs.650000

4)

Rs.800000

iv
at
e

1)

Li
m

ite
d

Question: 107
Choose the correct alternative.
Rs.3650000 is divided among 4 CFA's, 3 MBA's and 5 CA's such that 3 CA's get as much as 2 MBA's and 3 CFA's as much as 2 CA's.
Find the share of an MBA.

ng

es

ou

rc

es

Pr

Explanation:

1)

40 min

2)

37 min

3)

Le

ar

ni

Question: 108
Choose the correct alternative.
A monkey climbs 6 metres and falls 3 metres while climbing the walls of a well, in alternate minutes. Find the time taken to climb the wall
of 60 metres height.

IM

38 min

4)

39 min

Explanation:
Monkey climbs 6 m and descends 3 m in alternate minutes; hence in a set of 2 minutes he will be effectively climbing 3 m. Thus in 36
minutes i.e 18 sets it must be climbing 54 m. In 37thminute it will climb another 6 m and will finish climbing. Hence, [2].
Question: 109
Choose the correct alternative.

IMS Learning Resources Pvt.Ltd.,Mumbai.All copyrights to this material vestswith IMS Learning Resources Pvt.Ltd.
No part of this materials either in part oras a whole shall be copied,printed,electronically reproduced,sold or distributed without the written
consent of IMS Learing Resources Pvt.Ltd.and any such violation would entail initiation of suitable legal proceedings.

Copyright

Login ID:8FA03376/Student Name:ANSHULGARG/Overall Score:0

Three dice are rolled. The number of possible outcomes in which at least one die shows 5 is:
1)

215

2)

36

3)

125

4)

91

Li
m

ite
d

Explanation:

3:1

3)

5:4

4)

7:1

Pr

2)

es

4:7

rc

1)

iv
at
e

Question: 110
Choose the correct alternative.
A fruit-seller bought 20 dozen bananas for Rs.400, mixed with some damaged bananas whose price was Rs.10 per dozen. He sold 30
dozen bananas for Rs.900. In order to get a 60% profit, find the mixed proportion of fresh and damaged bananas he would have to sell.

Le

ar

ni

ng

es

ou

Explanation:

IM

Question: 111
Refer to the data below and answer the questions thatfollow.

IMS Learning Resources Pvt.Ltd.,Mumbai.All copyrights to this material vestswith IMS Learning Resources Pvt.Ltd.
No part of this materials either in part oras a whole shall be copied,printed,electronically reproduced,sold or distributed without the written
consent of IMS Learing Resources Pvt.Ltd.and any such violation would entail initiation of suitable legal proceedings.

Copyright

ar

ni

ng

es

ou

rc

es

Pr

iv
at
e

Li
m

ite
d

Login ID:8FA03376/Student Name:ANSHULGARG/Overall Score:0

Le

If the number of units of electricity consumed in March 1999 is 30% of the entire year, what is the ratio of the number of units of electricity
consumed in the year 2000 to the number of units of electricity consumed in the year 1999, if 50% for the year 2000 was consumed in
March?

IM

1)
2)

3)
4)

None of these

Explanation:

IMS Learning Resources Pvt.Ltd.,Mumbai.All copyrights to this material vestswith IMS Learning Resources Pvt.Ltd.
No part of this materials either in part oras a whole shall be copied,printed,electronically reproduced,sold or distributed without the written
consent of IMS Learing Resources Pvt.Ltd.and any such violation would entail initiation of suitable legal proceedings.

Copyright

ite
d

Login ID:8FA03376/Student Name:ANSHULGARG/Overall Score:0

Li
m

Question: 112
Refer to the data below and answer the questions thatfollow.

iv
at
e

What is the ratio of the average value of the highest number of units recorded in a day for the month of March to the average value of the
total number of units consumed in the month of March?
1)

Pr

2)

es

3)

rc

4)

ar

ni

ng

es

ou

Explanation:

Le

Question: 113
Refer to the data below and answer the questions that follow.

IM

What will be the value of the highest electricity consumed in any one day of March in the year 2005, if it increases from the year 2004 at
the same rate at which it grew from 2002 to 2003?
1)

36

2)

42

3)

32

4)

46

Explanation:

IMS Learning Resources Pvt.Ltd.,Mumbai.All copyrights to this material vestswith IMS Learning Resources Pvt.Ltd.
No part of this materials either in part oras a whole shall be copied,printed,electronically reproduced,sold or distributed without the written
consent of IMS Learing Resources Pvt.Ltd.and any such violation would entail initiation of suitable legal proceedings.

Copyright

Login ID:8FA03376/Student Name:ANSHULGARG/Overall Score:0

ite
d

Question: 114
Refer to the data below and answer the questions thatfollow.

2000-2001

2)

2002-2003

3)

2003-2004

4)

2001-2002

iv
at
e

1)

Li
m

During which year was the percentage change in electricity consumed in the month of March the highest?

rc

ou

Question: 115
Refer to the data below and answer the questions thatfollow.

es

Pr

Explanation:

2)

1.1 units

3)

1.6 units

4)

1.4 units

1.3 units

ng

1)

es

What is the average amount of electricity consumed in the 30 days of March 2002 other than the day on which highest electricity was
consumed?

Le

ar

ni

Explanation:

IM

Question: 116
Choose the correct alternative.

1)

0.80

2)

0.75

IMS Learning Resources Pvt.Ltd.,Mumbai.All copyrights to this material vestswith IMS Learning Resources Pvt.Ltd.
No part of this materials either in part oras a whole shall be copied,printed,electronically reproduced,sold or distributed without the written
consent of IMS Learing Resources Pvt.Ltd.and any such violation would entail initiation of suitable legal proceedings.

Copyright

Login ID:8FA03376/Student Name:ANSHULGARG/Overall Score:0

3)

0.25

4)

0.6

iv
at
e

Li
m

ite
d

Explanation:

10

3)

13.9

4)

8.7

rc

2)

ou

12

es

1)

es

Pr

Question: 117
Choose the correct alternative.
Find the approximate value of

Le

ar

ni

ng

Explanation:

IM

Question: 118
Find the value from the given options that would replace the question mark (?).

1)

15

2)

3)

10

4)

25

IMS Learning Resources Pvt.Ltd.,Mumbai.All copyrights to this material vestswith IMS Learning Resources Pvt.Ltd.
No part of this materials either in part oras a whole shall be copied,printed,electronically reproduced,sold or distributed without the written
consent of IMS Learing Resources Pvt.Ltd.and any such violation would entail initiation of suitable legal proceedings.

Copyright

Login ID:8FA03376/Student Name:ANSHULGARG/Overall Score:0

ite
d

Explanation:

2)

45

3)

47

4)

49

iv
at
e

38

Pr

1)

Li
m

Question: 119
Find the value from the given options that would replace the question mark (?).

ou

rc

es

Explanation:

25

3)

33

4)

30

ng

2)

ni

18

ar

1)

es

Question: 120
Find the value from the given options that would replace the question mark (?).

IM

Le

Explanation:

IMS Learning Resources Pvt.Ltd.,Mumbai.All copyrights to this material vestswith IMS Learning Resources Pvt.Ltd.
No part of this materials either in part oras a whole shall be copied,printed,electronically reproduced,sold or distributed without the written
consent of IMS Learing Resources Pvt.Ltd.and any such violation would entail initiation of suitable legal proceedings.

Copyright

Li
m

ite
d

Login ID:8FA03376/Student Name:ANSHULGARG/Overall Score:0

ni

ng

es

ou

rc

es

Pr

iv
at
e

Question: 121
Refer to the data below and answer the questions that follow.

Which month shows highest overall sales and how much?


$33700 Jan

2)

$29100 Feb

3)

$28700 Feb

4)

$35600 Jan

Le

ar

1)

IM

Explanation:
Greatest overall sales value is shown in month of January i.e., $(13,300 + 11,000 + 9400)= $ 33,700. Hence, [1].
Question: 122
Refer to the data below and answer the questions that follow.
Which month shows highest difference in sales of sofa and table and how much?
1)

Feb $2600

IMS Learning Resources Pvt.Ltd.,Mumbai.All copyrights to this material vestswith IMS Learning Resources Pvt.Ltd.
No part of this materials either in part oras a whole shall be copied,printed,electronically reproduced,sold or distributed without the written
consent of IMS Learing Resources Pvt.Ltd.and any such violation would entail initiation of suitable legal proceedings.

Copyright

Login ID:8FA03376/Student Name:ANSHULGARG/Overall Score:0

2)

Feb $2300

3)

April $4000

4)

April $3600

Explanation:
Greatest difference in sales of sofas and tables in month of April i.e., $12300 $8300 = $4000. Hence, [3].

$9600

2)

$7400

3)

$8500

4)

$10000

Li
m

1)

ite
d

Question: 123
Refer to the data below and answer the questions that follow.
What is the approximate monthly average value of sales of tables during over the entire period?

Pr

iv
at
e

Explanation:

rc

es

Question: 124
Refer to the data below and answer the questions that follow.

2)

Mar

3)

May

4)

June

es

Feb

1)

ou

Of the following, which month shows minimum percentage change in sales over the previous month for sofas?

IM

Le

ar

ni

ng

Explanation:

Question: 125
Refer to the data below and answer the questions that follow.
In the given period, during how many months has the sales of sofas been more than twice the sales of cupboards?

IMS Learning Resources Pvt.Ltd.,Mumbai.All copyrights to this material vestswith IMS Learning Resources Pvt.Ltd.
No part of this materials either in part oras a whole shall be copied,printed,electronically reproduced,sold or distributed without the written
consent of IMS Learing Resources Pvt.Ltd.and any such violation would entail initiation of suitable legal proceedings.

Copyright

Login ID:8FA03376/Student Name:ANSHULGARG/Overall Score:0

1)

2)

3)

4)

Explanation:
In no month has the sales of sofas been more than twice the sales of cuboards. Hence, [4].

Li
m

ite
d

Question: 126
Refer to the data below and answer the questions.
There are three bags containing certain number of cards. Bag I contains 12 red cards and 7 black cards. Bag II contains 8 red cards and
13 black cards. Bag III contains 6 red cards and 9 black cards. Also of the total cards in Bag I, Bag II and Bag III 9, 6 and 8 cards are face
cards respectively.

iv
at
e

If two cards are picked at random from each of the bags, then what is the probability that exactly five of these cards are red cards?
1)

es

Pr

2)

ou

rc

3)

es

4)

ar

ni

ng

Explanation:

Le

Question: 127
Refer to the data below and answer the questions.

59/133

IM

1)

If one card is picked at random from each of the bags then what is the probability that exactly 1 of the three cards is a face card?

2)

127/133

3)

181/399

4)

57/133

Explanation:

IMS Learning Resources Pvt.Ltd.,Mumbai.All copyrights to this material vestswith IMS Learning Resources Pvt.Ltd.
No part of this materials either in part oras a whole shall be copied,printed,electronically reproduced,sold or distributed without the written
consent of IMS Learing Resources Pvt.Ltd.and any such violation would entail initiation of suitable legal proceedings.

Copyright

Li
m

ite
d

Login ID:8FA03376/Student Name:ANSHULGARG/Overall Score:0

A-Rs.40 and B-Rs.10

2)

A-Rs.2 and B-Rs.5

3)

A-Rs.8 and B-Rs.2

4)

A-Rs.24 and B-Rs.6

es

Pr

1)

iv
at
e

Question: 128
Choose the correct alternative.
A and B play a game of tossing a coin in which one person wins and the other loses. Whoever wins gets Rs.10 and loser has to give out
Rs.5. They settle the account at the end of the game. They played ten games and at the end of it the ratio of the money won by A and B is
4 : 1. How much did each of them win?

ou

rc

Explanation:
In each game, the total money earned by both the players is Rs.5.At the end of 10 games the money earned by both the players will be
Rs.50.Therefore A gets Rs.40 and B gets Rs.10. Hence, [1].

10.8 cm

3)

11.2 cm

4)

10.4 cm

ng

2)

ni

9.8 cm

ar

1)

es

Question: 129
Choose the correct alternative.
Metallurgists melted three cubes whose edges are 3 cm, 4 cm, and 5 cm respectively to form a bigger cube. Find the diagonal of the
bigger cube.

IM

Le

Explanation:

Question: 130
Choose the correct alternative.
The word ACCIDENT is written on paper and each letter of the word is cut and put in a box. If three pieces of papers are taken at random
from this box one after other and kept on a table in the same order, what is the probability that the word so formed is CAT or CET?
1)

IMS Learning Resources Pvt.Ltd.,Mumbai.All copyrights to this material vestswith IMS Learning Resources Pvt.Ltd.
No part of this materials either in part oras a whole shall be copied,printed,electronically reproduced,sold or distributed without the written
consent of IMS Learing Resources Pvt.Ltd.and any such violation would entail initiation of suitable legal proceedings.

Copyright

Login ID:8FA03376/Student Name:ANSHULGARG/Overall Score:0

2)

3)

4)

Li
m

ite
d

Explanation:

ni

ng

es

ou

rc

es

Pr

iv
at
e

Question: 131
Refer to the table below and answer the questions that follow.

What was the decrease in total production from the year 2003 to the year 2004?
5 million tonnes

2)

15 million tonnes

3)

45 million tonnes

4)

65 million tonnes

Le

ar

1)

IM

Explanation:
Total production in 2003
= 40 + 40 + 60 + 30 + 35 + 15 + 40 + 40 + 30 + 30 = 360 million tonnes
Total production in 2004
= 30 + 60 + 40 + 25 + 50 + 30 + 40 + 30 + 20 + 20 = 345 million tonnes
Hence, decrease was 15 million tonnes.
Hence, [2].
Question: 132

IMS Learning Resources Pvt.Ltd.,Mumbai.All copyrights to this material vestswith IMS Learning Resources Pvt.Ltd.
No part of this materials either in part oras a whole shall be copied,printed,electronically reproduced,sold or distributed without the written
consent of IMS Learing Resources Pvt.Ltd.and any such violation would entail initiation of suitable legal proceedings.

Copyright

Login ID:8FA03376/Student Name:ANSHULGARG/Overall Score:0

Refer to the table below and answer the questions that follow.
What was the increase in total sales from the year 2001 to the year 2002?
1)

13 million tonnes

2)

25 million tonnes

3)

33 million tonnes

4)

45 million tonnes

iv
at
e

Li
m

ite
d

Explanation:
Total sales in 2001
= 5 + 50 + 15 + 15 + 40 + 30 + 15 + 20 + 10 + 60 = 260 million tonnes
Total sales in 2002
= 15 + 19 + 40 + 25 + 35 + 19 + 20 + 45 + 30 + 25 = 273 million tonnes
Hence, increase in sales is 13 million tonnes.
Hence, [1].
Question: 133
Refer to the table below and answer the questions that follow.

2)

140 million tonnes

3)

160 million tonnes

4)

180 million tonnes

es

120 million tonnes

rc

1)

Pr

What is the total sales for factory 5 across all five years?

es

ou

Explanation:
The total sales for factory 5 across the five years = 40 + 35 + 25 + 45 + 15
= 160 million tonnes. Hence, [3].

Question: 134
Refer to the table below and answer the questions that follow.

ng

What is the ratio of total sales to total production for factory 7 between the years 2001-2003 (both years inclusive)?

ar

ni

1)

Le

2)

IM

3)

4)

Explanation:

IMS Learning Resources Pvt.Ltd.,Mumbai.All copyrights to this material vestswith IMS Learning Resources Pvt.Ltd.
No part of this materials either in part oras a whole shall be copied,printed,electronically reproduced,sold or distributed without the written
consent of IMS Learing Resources Pvt.Ltd.and any such violation would entail initiation of suitable legal proceedings.

Copyright

Login ID:8FA03376/Student Name:ANSHULGARG/Overall Score:0

2)

52.3%

3)

54.5%

4)

56.8%

Li
m

48.5%

iv
at
e

1)

ite
d

Question: 135
Refer to the table below and answer the questions that follow.
What percentage of production was sales for factories 1, 2 and 3 combined for the years 2001-2003?

ou

rc

es

Pr

Explanation:

2)

[2]

3)

[3]
[4]

IM

4)

Le

[1]

1)

ar

ni

ng

es

Question: 136
Each question is followed by two statements I and II. Answer each question using the following instructions.
Mark [1], if the question can be answered by using only one statement alone.
Mark [2], if the question can be answered by using both the statements together, but cannot be answered by either of the statements
alone.
Mark [3], if the question can be answered by either of the statements alone.
Mark [4], if more data is required to answer the question.
Did each of the 20 people carry atleast 8 kg of luggage?
I. One person carried twice as much of each of the other nineteen people.
II. The average weight carried by the twenty people is 8.25 kg.

Explanation:
Statement I:
If each of the other 19 people carried x of kg luggage, then the person carried 2x kg of
luggage.
Statement II:
The total weight carried by all 20 people is 8.25 20 = 165 kg.
Both the statements individually are not sufficient to answer the question. But if we

IMS Learning Resources Pvt.Ltd.,Mumbai.All copyrights to this material vestswith IMS Learning Resources Pvt.Ltd.
No part of this materials either in part oras a whole shall be copied,printed,electronically reproduced,sold or distributed without the written
consent of IMS Learing Resources Pvt.Ltd.and any such violation would entail initiation of suitable legal proceedings.

Copyright

Login ID:8FA03376/Student Name:ANSHULGARG/Overall Score:0

combine both the statements we get the answer.


Hence, [2].

2)

[2]

3)

[3]

4)

[4]

es

Explanation:
I: As the intersection of line L and P is a single point, the line L does not lie in the
plane P.
II: We cannot determine whether the line L lies in the plane P or not. Hence, [1].

iv
at
e

[1]

Pr

1)

Li
m

ite
d

Question: 137
Each question is followed by two statements I and II. Answer each question using the following instructions.
Mark [1], if the question can be answered by using only one statement alone.
Mark [2], if the question can be answered by using both the statements together, but cannot be answered by either of the statements
alone.
Mark [3], if the question can be answered by either of the statements alone.
Mark [4], if more data is required to answer the question.
There is a line L and a plane P. Does plane P contain the line L?
I. L and P intersect at a single point O.
II. L is paralled to line X which is parallel to plane P.

[2]

3)

[3]

4)

[4]

ni

2)

ar

[1]

Le

1)

ng

es

ou

rc

Question: 138
Each question is followed by two statements I and II. Answer each question using the following instructions.
Mark [1], if the question can be answered by using only one statement alone.
Mark [2], if the question can be answered by using both the statements together, but cannot be answered by either of the statements
alone.
Mark [3], if the question can be answered by either of the statements alone.
Mark [4], if more data is required to answer the question.

IM

Explanation:
From statement I,
we do not know whether AC is also a perpendicular bisector of BD. Hence, we cannot
answer the question.
Statement II:
If ABCD is a rhombus, then none of its angles will be 90o. Hence, ABCD is not a rhombus.
Hence,[1].
Question: 139
Each question is followed by two statements I and II. Answer each question using the following instructions.

IMS Learning Resources Pvt.Ltd.,Mumbai.All copyrights to this material vestswith IMS Learning Resources Pvt.Ltd.
No part of this materials either in part oras a whole shall be copied,printed,electronically reproduced,sold or distributed without the written
consent of IMS Learing Resources Pvt.Ltd.and any such violation would entail initiation of suitable legal proceedings.

Copyright

Login ID:8FA03376/Student Name:ANSHULGARG/Overall Score:0

[1]

2)

[2]

3)

[3]

4)

[4]

Li
m

1)

ite
d

Mark [1], if the question can be answered by using only one statement alone.
Mark [2], if the question can be answered by using both the statements together, but cannot be answered by either of the statements
alone.
Mark [3], if the question can be answered by either of the statements alone.
Mark [4], if more data is required to answer the question.
What is Ram's age 1 year hence?
I. His age was twice that of Shyam's age a year ago.
II. Shyam is seventeen years younger to Ram's twin brother.

rc

es

Pr

iv
at
e

Explanation:

Rs.28000

3)

Rs.29950

4)

Rs.31500

IM

Le

Explanation:

ng

2)

ni

Rs.33455

ar

1)

es

ou

Question: 140
Choose the correct alternative.

Question: 141
Choose the correct alternative.

IMS Learning Resources Pvt.Ltd.,Mumbai.All copyrights to this material vestswith IMS Learning Resources Pvt.Ltd.
No part of this materials either in part oras a whole shall be copied,printed,electronically reproduced,sold or distributed without the written
consent of IMS Learing Resources Pvt.Ltd.and any such violation would entail initiation of suitable legal proceedings.

Copyright

Login ID:8FA03376/Student Name:ANSHULGARG/Overall Score:0

If all possible 4-digit numbers formed using the digits 1, 2, 3, 4 without repetition are arranged in the ascending order, find the position of
the number 3412.
1)

16

2)

17

3)

14

4)

15

Li
m

ite
d

Explanation:
The number of 4 digit numbers having 1 or 2 in the first digit is
2 3! = 12. The numbers starting with 3 in the ascending order are 3124, 3142, 3214,
3241, 3412.
The position of 3412 is 17.
Hence, [2].

Pr

iv
at
e

Question: 142
Choose the correct alternative.
Fifty balls in a basket are numbered from 1 to 50. If a ball is drawn at random from the basket what are the chances that it will have a
number divisible by 4 or 7?

es

1)

ou

rc

2)

es

3)

4)

IM

Le

ar

ni

ng

Explanation:

Question: 143
Choose the correct alternative.

IMS Learning Resources Pvt.Ltd.,Mumbai.All copyrights to this material vestswith IMS Learning Resources Pvt.Ltd.
No part of this materials either in part oras a whole shall be copied,printed,electronically reproduced,sold or distributed without the written
consent of IMS Learing Resources Pvt.Ltd.and any such violation would entail initiation of suitable legal proceedings.

Copyright

1)

344

2)

352

3)

362

4)

364

ite
d

Login ID:8FA03376/Student Name:ANSHULGARG/Overall Score:0

iv
at
e

Li
m

Explanation:

20

3)

25

4)

30

rc

2)

ou

10

es

1)

es

Pr

Question: 144
Choose the correct alternative.
A contractor employs 50 labourers to builds a wall in 50 days. However, after 25 days, he finds that only 40% of the work is complete.
How many more labourers need to be employed to complete the work in 50 days?

IM

Le

ar

ni

ng

Explanation:

Question: 145
Choose the correct alternative.
Sides of two congruent squares A and B each having side of length 25 cm, are increased by 6% and 10% respectively, what will be the
difference between their areas?

IMS Learning Resources Pvt.Ltd.,Mumbai.All copyrights to this material vestswith IMS Learning Resources Pvt.Ltd.
No part of this materials either in part oras a whole shall be copied,printed,electronically reproduced,sold or distributed without the written
consent of IMS Learing Resources Pvt.Ltd.and any such violation would entail initiation of suitable legal proceedings.

Copyright

Login ID:8FA03376/Student Name:ANSHULGARG/Overall Score:0

1)

53 sq. cm.

2)

54 sq. cm.

3)

65 sq. cm.

4)

48 sq. cm.

Pr

rc

es

Question: 146
Refer to the data below and answer the questions that follow.
Mark [1]; if quantity in I is greater than that in II.
Mark [2]; if quantity in I is less than that in II.
Mark [3]; if quantity in I is equal to that in II.
Mark [4]; if the relationship between quantity in I and II cannot be determined.

iv
at
e

Li
m

ite
d

Explanation:

2)

[2]

3)

[3]

4)

[4]

[1]

ng

1)

es

ou

Shopkeeper A offers a discount of 25.5% on an item priced Rs.675. Shopkeeper B offers


successive discounts of 11% and 12% on an item priced at Rs.650.
I.Selling price of shopkeeper A.
II. Selling price of shopkeeper B.

IM

Le

ar

ni

Explanation:

Question: 147
Refer to the data below and answer the questions that follow.
Mark [1]; if quantity in I is greater than that in II.
Mark [2]; if quantity in I is less than that in II.
Mark [3]; if quantity in I is equal to that in II.
Mark [4]; if the relationship between quantity in I and II cannot be determined.

IMS Learning Resources Pvt.Ltd.,Mumbai.All copyrights to this material vestswith IMS Learning Resources Pvt.Ltd.
No part of this materials either in part oras a whole shall be copied,printed,electronically reproduced,sold or distributed without the written
consent of IMS Learing Resources Pvt.Ltd.and any such violation would entail initiation of suitable legal proceedings.

Copyright

1)

[1]

2)

[2]

3)

[3]

4)

[4]

ite
d

Login ID:8FA03376/Student Name:ANSHULGARG/Overall Score:0

es

Pr

iv
at
e

Li
m

Explanation:

[1]

2)

[2]

3)

[3]

4)

Le

1)

ar

ni

ng

es

ou

rc

Question: 148
Refer to the data below and answer the questions that follow.
Mark [1]; if quantity in I is greater than that in II.
Mark [2]; if quantity in I is less than that in II.
Mark [3]; if quantity in I is equal to that in II.
Mark [4]; if the relationship between quantity in I and II cannot be determined.

IM

[4]

Explanation:

IMS Learning Resources Pvt.Ltd.,Mumbai.All copyrights to this material vestswith IMS Learning Resources Pvt.Ltd.
No part of this materials either in part oras a whole shall be copied,printed,electronically reproduced,sold or distributed without the written
consent of IMS Learing Resources Pvt.Ltd.and any such violation would entail initiation of suitable legal proceedings.

Copyright

[2]

3)

[3]

4)

[4]

iv
at
e
Pr

rc

2)

ou

[1]

ng

Explanation:
One class monitor can be chosen in 15C1= 15 ways.
Out of 6, two balls can be chosen in 6C2= 15 ways.
Hence, [3].

es

1)

es

Question: 149
Refer to the data below and answer the questions that follow.
Mark [1]; if quantity in I is greater than that in II.
Mark [2]; if quantity in I is less than that in II.
Mark [3]; if quantity in I is equal to that in II.
Mark [4]; if the relationship between quantity in I and II cannot be determined.
I. Number of ways of choosing a class monitor from a class of 15 students.
II. Number of ways of choosing 2 balls from a box containing 6 balls.

Li
m

ite
d

Login ID:8FA03376/Student Name:ANSHULGARG/Overall Score:0

IM

Le

ar

ni

Question: 150
Refer to the data below and answer the questions that follow.
Mark [1]; if quantity in I is greater than that in II.
Mark [2]; if quantity in I is less than that in II.
Mark [3]; if quantity in I is equal to that in II.
Mark [4]; if the relationship between quantity in I and II cannot be determined.

1)

[1]

2)

[2]

3)

[3]

4)

[4]

Explanation:

IMS Learning Resources Pvt.Ltd.,Mumbai.All copyrights to this material vestswith IMS Learning Resources Pvt.Ltd.
No part of this materials either in part oras a whole shall be copied,printed,electronically reproduced,sold or distributed without the written
consent of IMS Learing Resources Pvt.Ltd.and any such violation would entail initiation of suitable legal proceedings.

Copyright

Login ID:8FA03376/Student Name:ANSHULGARG/Overall Score:0

ite
d

Question: 151
Each question consists of two capitalized words that have a certain relationship to each other, followed by 4 pairs of words. Choose the
pair that is related to each other in the same way as the capitalized pair.

vulture : vulpine

2)

egret : equine

3)

wolf : ursine

4)

cow : bovine

iv
at
e

1)

Li
m

CAT : FELINE

Pr

Explanation:
'Cat' belongs to the 'feline' group of animals. In the same way, a 'cow' belongs to the 'bovine' group of animals. Hence, [4].

es

Question: 152
Each question consists of two capitalized words that have a certain relationship to each other, followed by 4 pairs of words. Choose the
pair that is related to each other in the same way as the capitalized pair.

sextant : distance

3)

abacus : amount

4)

gyroscope : speed

ou

2)

es

telescope : size

1)

rc

COMPASS : DIRECTION

ni

ng

Explanation:
'Compass' is used by travelers to find their direction. A 'sextant' is an instrument for measuring the angular distance between two objects
and is especially used for determining latitudes at the sea. Hence, [2].

Le

ar

Question: 153
Each question consists of two capitalized words that have a certain relationship to each other, followed by 4 pairs of words. Choose the
pair that is related to each other in the same way as the capitalized pair.
PRUNE :PLUM
bird : feathers

2)

fruit : date

3)

1)

4)

raisin : grape

IM

fig : apricot

Explanation:
A 'prune' is a dried 'plum' and a 'raisin' is a dried 'grape'. Hence, [4].
Question: 154
Each question consists of two capitalized words that have a certain relationship to each other, followed by 4 pairs of words. Choose the
pair that is related to each other in the same way as the capitalized pair.

IMS Learning Resources Pvt.Ltd.,Mumbai.All copyrights to this material vestswith IMS Learning Resources Pvt.Ltd.
No part of this materials either in part oras a whole shall be copied,printed,electronically reproduced,sold or distributed without the written
consent of IMS Learing Resources Pvt.Ltd.and any such violation would entail initiation of suitable legal proceedings.

Copyright

Login ID:8FA03376/Student Name:ANSHULGARG/Overall Score:0

WORM : HOOK
1)

whale : dolphin

2)

fish : eye

3)

scapegoat : disorder

4)

cheese : mousetrap

ite
d

Explanation:
A 'worm' is used as bait in the 'hook' while fishing; in the same manner, 'cheese' is used as the bait in a 'mousetrap' to catch a mouse.
Hence, [4].

annihilate : crush

2)

chaos : order

3)

win : lose

4)

giggle : guffaw

Pr

1)

iv
at
e

Li
m

Question: 155
Each question consists of two capitalized words that have a certain relationship to each other, followed by 4 pairs of words. Choose the
pair that is related to each other in the same way as the capitalized pair.
SIMMER:BOIL

es

Explanation:
'Simmering' and 'boiling' are different degrees of heating; in the same manner 'giggling' and 'guffawing' are different degrees of laughter (a
guffaw is a burst of deep, loud, hearty laughter, while a giggle is a foolish or nervous laugh). Hence, [4].

es

ou

rc

Question: 156
In each of the following sentences, a part of the sentence or the entire sentence is underlined. Beneath each sentence, four different ways
of phrasing the underlined part are indicated. Choose the best alternative from among the four given options.

which has been translated into foreign languages worldwide, he hosts a radio programme and a television show, and has
been using them to argue against the age-old Russian literary mantra that man, in his heart of hearts, is inherently good.

2)

that have been translated into foreign languages worldwide, they host a radio programme and a television show, and are
using them to argue against the age-old Russian literary mantra that man, in his heart of hearts, is inherently good.

3)

which is translated into foreign languages worldwide, he hosts a radio programme and a television show, and is using them
to argue against the age-old Russian literary mantra that man, in his heart of hearts, is inherently good.

4)

that has been translated into foreign languages worldwide, he hosts a radio programme and a television show, and has
been using them to argue against the age-old Russian literary mantra that man, in his heart of hearts, are inherently good.

Le

ar

ni

ng

1)

IM

Explanation:
[4] is incorrect because there is a subject verb mismatch: 'man' is singular, so the verb should be 'is' and not 'are'. [3] does not have a
parallel construction. [2] also has a subject verb mismatch: 'fiction' is singular, so the verb should be 'has' and not 'have'. Hence, [1].
Question: 157
In each of the following sentences, a part of the sentence or the entire sentence is underlined. Beneath each sentence, four different ways
of phrasing the underlined part are indicated. Choose the best alternative from among the four given options.

IMS Learning Resources Pvt.Ltd.,Mumbai.All copyrights to this material vestswith IMS Learning Resources Pvt.Ltd.
No part of this materials either in part oras a whole shall be copied,printed,electronically reproduced,sold or distributed without the written
consent of IMS Learing Resources Pvt.Ltd.and any such violation would entail initiation of suitable legal proceedings.

Copyright

Login ID:8FA03376/Student Name:ANSHULGARG/Overall Score:0

1)

are one of the only surgical procedures which eagerly embrace the practise of self-diagnosis.

2)

is the only surgical procedure that eagerly embraces the practice of self-diagnosis.

3)

is the only surgical procedures which eagerly embraces the practice of self-diagnosis.

4)

is one of the only surgical procedures that eagerly embraces the practice of self-diagnosis.

ite
d

Explanation:
In statements [1] and [4], 'one of the only surgical procedures' is grammatically incorrect. In [3], there is no agreement between 'only',
'procedures' and 'embraces'. Hence, [2].

iv
at
e

Li
m

Question: 158
In each of the following sentences, a part of the sentence or the entire sentence is underlined. Beneath each sentence, four different ways
of phrasing the underlined part are indicated. Choose the best alternative from among the four given options.

It reflected not only a distorted view of a critical episode in U.S. foreign policy but also the rejection of important, negative
lessons that Americans later drew from their brief experiment in creating an overseas empire.

2)

It reflected not only a distorted view of a critical episode in U.S. foreign policy but the rejection of important, negative
lessons that Americans later drew from their brief experiment in creating an overseas empire.

3)

It reflected not only a distorted view of the critical episode in U.S. foreign policy but also the rejection of important, negative
lessons that Americans later draw from their brief experiment in the creation of an overseas empire.

4)

It reflected not only a distorted view of a critical episode of U.S. foreign policy but also the rejection of important, negative
lessons that Americans later drew from its brief experiment in creating an overseas empire.

ou

rc

es

Pr

1)

es

Explanation:
Sentence [2] is incorrect as 'not only' should always be followed by 'but also'. In sentence [3], 'draw' should be 'drew' as the sentence is in
past tense. In sentence [4], the pronoun 'its' should be 'their' as it refers to Americans which is plural. Hence, [1].

ni

ng

Question: 159
In each of the following sentences, a part of the sentence or the entire sentence is underlined. Beneath each sentence, four different ways
of phrasing the underlined part are indicated. Choose the best alternative from among the four given options.

of these tacit concepts reveal that notions of religious agency, despite important cultural differences, are very similar the
world over.

2)

of these tacit concepts reveals that notions of religious agency, despite important cultural differences, are very similar the
world over.

3)

of these tacit concepts reveals that notions of religious agency, despite important cultural differences, is very similar the
world over.

IM

Le

ar

1)

4)

of these tacit concepts reveal that notions of religious agency, despite important cultural differences, is very similar the
world over.

Explanation:
Sentence [1] is incorrect as the verb 'reveal' should be 'reveals' as its subject is 'systematic investigation', which is singular. Sentence [4]
also has two instances of subject-verb mismatch. [3] is also incorrect as 'important cultural differences' is plural, hence the verb 'are'
should be used with it. Hence, [2].

IMS Learning Resources Pvt.Ltd.,Mumbai.All copyrights to this material vestswith IMS Learning Resources Pvt.Ltd.
No part of this materials either in part oras a whole shall be copied,printed,electronically reproduced,sold or distributed without the written
consent of IMS Learing Resources Pvt.Ltd.and any such violation would entail initiation of suitable legal proceedings.

Copyright

Login ID:8FA03376/Student Name:ANSHULGARG/Overall Score:0

Question: 160
In each of the following sentences, a part of the sentence or the entire sentence is underlined. Beneath each sentence, four different ways
of phrasing the underlined part are indicated. Choose the best alternative from among the four given options.

was exceptional among the leading economists in understanding the need to confront grossmisconceptions about
economics in the general public, including the so-called educated public.

2)

is exceptional between leading economists in understanding the need to confront gross misconceptions about economics of
the general public, including the so-called educated public.

3)

is exceptional among the leading economists in understanding the need for confronting gross misconceptions of economics
of the general public, including the so-called educated public.

4)

was exceptional among the leading economists for understanding the need to confront gross misconceptions about
economics in the general public, including the so-called educated public.

Li
m

ite
d

1)

iv
at
e

Explanation:
Option [2] is incorrect as the word 'among' should be used instead of 'between'. In option [4], the preposition 'for' should be replaced by
'in'. One holds misconceptions about something, so 'of' is a wrong preposition. This error is present in [3]. Hence, [1].

es

Pr

Question: 161
In each of the sentences, parts of the sentences are left blank. Beneath each sentence, four different ways of completing the sentence are
indicated. Choose the word or set of words that best fits the meaning of the sentence as a whole.
His __________ ability to give the most commonplace looks a _________ twist a thermal undershirt made of cashmere, or cropped
cargo pants turned into lavish tuxedo pants has made him one of the most carefully watched and desired designers in the business.
uncanny .......... smart

2)

superficial ..........natty

3)

exceptional ..........luxurious

4)

bizarre ..........whacky

es

ou

rc

1)

ng

Explanation:
Look at what he is doing to his clothes. He is giving casual clothes a lavish look. Only 'luxurious' fits in best in the second blank. [1] does
not fit in because the word 'uncanny' has a mysterious connotation. Hence, [3].

Le

ar

ni

Question: 162
In each of the sentences, parts of the sentences are left blank. Beneath each sentence, four different ways of completing the sentence are
indicated. Choose the word or set of words that best fits the meaning of the sentence as a whole.
Choosing to work in his own vein, extracting a __________ ore instead of _________ from one popular and profitable mine-head to
another, he has nevertheless experimented with diverse media platforms.
consistent ..........flitting

2)

tender ..........contaminating

3)

1)

4)

brilliant ..........borrowing

IM

incongruous ..........varying

Explanation:
Options [2] and [3] do not fit in the blanks from a grammatical point of view. Between [1] and [4], [1] fits in logically as well as idiomatically.
Hence, [1].
Question: 163
In each of the sentences, parts of the sentences are left blank. Beneath each sentence, four different ways of completing the sentence are

IMS Learning Resources Pvt.Ltd.,Mumbai.All copyrights to this material vestswith IMS Learning Resources Pvt.Ltd.
No part of this materials either in part oras a whole shall be copied,printed,electronically reproduced,sold or distributed without the written
consent of IMS Learing Resources Pvt.Ltd.and any such violation would entail initiation of suitable legal proceedings.

Copyright

Login ID:8FA03376/Student Name:ANSHULGARG/Overall Score:0

indicated. Choose the word or set of words that best fits the meaning of the sentence as a whole.
No longer ____________ in the face of fraud, accounting firms are __________ on new US laws that give them real clout.
1)

helpless ..........dependent

2)

obstinate ..........flexible

3)

frail ..........thriving

4)

stagnant ..........booming

Li
m

ite
d

Explanation:
The new US laws are giving the accounting firms real clout. This means that they are no longer 'helpless' or 'frail' in the face of fraud. This
narrows down our answer options to [1] and [3]. However, since the new US laws are giving these firms real clout, we can infer that the
accounting firms would be 'thriving' on them. Hence, [3].

2)

contradictory ......... superior

3)

superficial .........all-encompassing

4)

opposites .........subsuming

Pr

complimentary ......... overpowering

es

1)

iv
at
e

Question: 164
In each of the sentences, parts of the sentences are left blank. Beneath each sentence, four different ways of completing the sentence are
indicated. Choose the word or set of words that best fits the meaning of the sentence as a whole.
Instead of seeing the worldly and the transcendent as _____________, it might be more fruitful to regard the second set as _________,
including and exceeding the first.

ou

rc

Explanation:
According to the author, it would be fruitful to regard the second set (transcendent) as including and exceeding the first (worldly). This
means that we should not see them as 'contradictory' or 'opposites'. This narrows down our answer options to [2] and [4]. But since the
second set is including and exceeding the first, we can conclude that it is 'subsuming'. Hence, [4].

ng

es

Question: 165
In each of the sentences, parts of the sentences are left blank. Beneath each sentence, four different ways of completing the sentence are
indicated. Choose the word or set of words that best fits the meaning of the sentence as a whole.
Earthy in outlook but ____________ in style, the Punjabi-origin farmers ___________ the Great American Dream by dint of hard work and
resilience.
rustic ..........follow

2)

flexible ..........execute

3)

diverse ..........appreciate

4)

sophisticated ........... realize

Le

ar

ni

1)

IM

Explanation:
The hint lies in the use of the term 'but'. This means that the term in the first blank must be somewhat an opposite of what is mentioned in
the beginning of the sentence. So 'sophisticated' seems to fit in best in the context and one 'realizes' a dream. Hence, [4].
Question: 166
Find the odd one out.
1)

audacious

2)

foolhardy

3)

reckless

4)

valiant

IMS Learning Resources Pvt.Ltd.,Mumbai.All copyrights to this material vestswith IMS Learning Resources Pvt.Ltd.
No part of this materials either in part oras a whole shall be copied,printed,electronically reproduced,sold or distributed without the written
consent of IMS Learing Resources Pvt.Ltd.and any such violation would entail initiation of suitable legal proceedings.

Copyright

Login ID:8FA03376/Student Name:ANSHULGARG/Overall Score:0

Explanation:
Though all the words mean brave or bold, options [1] to [3] are used mostly in a negative sense, while [4] has only a positive connotation.
Hence, [4].

1)

rococo

2)

haiku

3)

ballad

4)

lyric

ite
d

Question: 167
Find the odd one out.

Li
m

Explanation:
'Rococo' is a style of art. All others are types of poems. Hence, [1].

smash

2)

shatter

3)

spatter

4)

split

Pr

1)

iv
at
e

Question: 168
Find the odd one out.

es

Explanation:
[1], [2] and [4] are words that imply forceful action. 'Spatter' means to 'splash'. Hence, [3].

chain

2)

pedal

3)

saddle

4)

windscreen

1)

es

ou

rc

Question: 169
Find the odd one out.

2)

scholar

3)

sermon

4)

ar

homily

Le

1)

Question: 170
Find the odd one out.

ni

ng

Explanation:
'Windscreen' is a part of a car. All others are parts of a bicycle. Hence, [4].

IM

lecture

Explanation:
All the words except 'scholar' mean 'sermon' or 'lecture'. Hence, [2].
Question: 171
In the sentences given below, certain parts have been underlined. Mark the part that has an error in it. If there is no error, mark [4].

IMS Learning Resources Pvt.Ltd.,Mumbai.All copyrights to this material vestswith IMS Learning Resources Pvt.Ltd.
No part of this materials either in part oras a whole shall be copied,printed,electronically reproduced,sold or distributed without the written
consent of IMS Learing Resources Pvt.Ltd.and any such violation would entail initiation of suitable legal proceedings.

Copyright

1)

[1]

2)

[2]

3)

[3]

4)

[4]

ite
d

Login ID:8FA03376/Student Name:ANSHULGARG/Overall Score:0

Li
m

Explanation:
There is a tense error in part [3] the correct form of the verb should be simply 'would answer', not 'would have answered' (in keeping with
'offered' earlier in the sentence). Hence, [3].

[2]

3)

[3]

4)

[4]

ou

2)

es

[1]

1)

rc

es

Pr

iv
at
e

Question: 172
In the sentences given below, certain parts have been underlined. Mark the part that has an error in it. If there is no error, mark [4].

ng

Explanation:
In part [3], the words 'from' and 'form' are mixed up. Hence, [3].

IM

Le

ar

ni

Question: 173
In the sentences given below, certain parts have been underlined. Mark the part that has an error in it. If there is no error, mark [4].

1)

[1]

2)

[2]

3)

[3]

IMS Learning Resources Pvt.Ltd.,Mumbai.All copyrights to this material vestswith IMS Learning Resources Pvt.Ltd.
No part of this materials either in part oras a whole shall be copied,printed,electronically reproduced,sold or distributed without the written
consent of IMS Learing Resources Pvt.Ltd.and any such violation would entail initiation of suitable legal proceedings.

Copyright

Login ID:8FA03376/Student Name:ANSHULGARG/Overall Score:0

4)

[4]

Explanation:
There is no error in the given sentence. Hence, [4].

2)

[2]

3)

[3]

4)

[4]

Li
m

[1]

iv
at
e

1)

ite
d

Question: 174
In the sentences given below, certain parts have been underlined. Mark the part that has an error in it. If there is no error, mark [4].

Explanation:
The correct comparative form of 'soon' is 'sooner', not 'more soon'. Hence, [1].

[2]

3)

[3]

4)

[4]

ou

2)

es

[1]

1)

rc

es

Pr

Question: 175
In the sentences given below, certain parts have been underlined. Mark the part that has an error in it. If there is no error, mark [4].

ng

Explanation:
The correct preposition in part [2] should be 'of' not 'for'. Hence, [2].

ar

ni

Question: 176
Read the passage below and answer the questions that follow.

IM

Le

In the beginning there was only darkness. For many millions of years this darkness remained. There were no stars, no sun, and no earth.
But one day, something very special happened. The darkness created light. It was a very small amount of light but it was just enough. The
dark became the husband of the light. After a long while, both the light and the dark became bored.
The light began to insult the dark and the dark replied with equally harsh insults.
"You are not as beautiful as I!" said the light.
"Ha! You are much uglier than I!" said the dark.
Eventually, they began to fight. None could triumph over the other, however, because they both were equal in power. Dark had a little bit
more strength but light was a little bit more cunning. They continued to fight for a very long time, until one day, the light was tired and she
called to the dark:
"We must create beings that will give more meaning to our lives for we must live together for eternity."
The dark agreed and so it was settled. They would conceive two new beings. One would be female, the other would be male and the two
siblings would be fertile and give birth to the universe. The light gave birth to a god, the first god, and his name was Thrakath. He was the
creator of the universe and the son of the light and the dark. The light also gave birth to Tria and she was the creator of the universe, the

IMS Learning Resources Pvt.Ltd.,Mumbai.All copyrights to this material vestswith IMS Learning Resources Pvt.Ltd.
No part of this materials either in part oras a whole shall be copied,printed,electronically reproduced,sold or distributed without the written
consent of IMS Learing Resources Pvt.Ltd.and any such violation would entail initiation of suitable legal proceedings.

Copyright

Login ID:8FA03376/Student Name:ANSHULGARG/Overall Score:0

Le

ar

ni

ng

es

ou

rc

es

Pr

iv
at
e

Li
m

ite
d

daughter of the light and the dark and the wife of Thrakath.
Both Thrakath and Tria lived together within the light and the dark for a long time creating the universe within their minds. After they had
finished thinking, they decided to implement their plans. First they created the stars. There were many stars throughout the universe and
both Thrakath and Tria were pleased. But the stars eventually became troublesome, so they decided to create the sun.
The sun became the king of the universe and he was very bright, so he could be seen from far and wide. The stars feared him and so
again began to behave in an appropriate fashion. He also was very hot and warmed the once cold universe so that life would be possible.
Then, Thrakath and Tria decided to make planets. They made eight planets, one every month for eight months. These planets were much
smaller than the stars and they worshipped the sun and revolved round him. They waited three more months until they created the ninth
planet. This planet they named Earth and decided to make her special. The other planets became jealous but the ever present sun
discouraged them from displaying unrest.
The Earth became extremely weary of her life so Thrakath and Tria gave her a friend who would be her eternal companion named Moon.
Moon and Earth became good friends and they stayed together. Thrakath and Tria were very pleased with what they had created but they
were not quite finished.
They were tired of creating, however, and so they created an earth-god name Terra who would develop and populate the earth. Terra
wasted no time in getting to work. First he created water and then he created land. When he had finished, he thought to himself how very
beautiful Earth was and he was also pleased. Then he reached into his pockets and took out some seeds. Terra proceeded to plant these
seeds in the land. They were all different types of seeds and from them sprang many different trees. Some bore fruit and others didn't.
Terra thought to himself that the plants were nice but were not interesting enough, so he planted special seeds that would grow into
animals. Only two of each type of animals were created; but they mated and reproduced and populated the earth. He then thought to
himself that the animals were interesting but were not very smart. He wanted to create individuals who, like himself, could also create. So
after much thought, he created people.
The people were very smart. They learned much by just observing the environment around them. They used the plants to create tools and
shelter for themselves. They also used plants for food. They killed some of the animals and used them for clothing and also for food. Terra
was ecstatic at what he had created and bragged about the people to Thrakath and Tria who were interested.
For many hundreds of years the people were good, taking only what they needed from the earth and obeying the wishes of Terra, the
earth-god. They always remembered to worship him for they owed him much. People loved Terra and each other and were happy. They
reproduced and soon populated the earth along with the animals.
But then humans started to do bad things. They began to kill each other, waste the food, and disobey Terra. They no longer worshipped
him and many forgot him altogether. The people thought that they could live well without respecting the earth-god. Terra, the earth-god,
became very displeased and so he created a great and terrible monster to deal with the people. The monster was named Dragga and he
was a violent and terrifying beast. He swept down upon the earth with great fury and devoured as many people as his immense belly
could hold. Those that he could not eat, he grabbed with his enormous claws and snapped in half.
The people were horrified and called out to Terra to have mercy and save them. Terra heard their cries and had pity for them so he
created a great earth warrior named Hercules to slay the beast. Hercules had superhuman strength and possessed a sharp, doubleedged sword. He travelled many miles to find Dragga and when he found the monster he immediately slayed him. The people were
grateful to Terra for sending them such a fearless warrior to slay Dragga and made Hercules their king. With Hercules as their king, the
people of the earth once again became prosperous and the world was returned to order.

As soon as Thrakath and Tria were born, they created the universe.

IM

1)

According to the passage, which of the following statements is not true?

2)

The sun was created to keep control over the stars.

3)

Animals were created on the earth as plants were not fascinating enough.

4)

Dragga was the name of the monster whom the earth-god created.

Explanation:
Refer to paragraph 8. Thrakath and Tria first created the universe in their minds. It was only later that they implemented their plans.
Hence, [1].

IMS Learning Resources Pvt.Ltd.,Mumbai.All copyrights to this material vestswith IMS Learning Resources Pvt.Ltd.
No part of this materials either in part oras a whole shall be copied,printed,electronically reproduced,sold or distributed without the written
consent of IMS Learing Resources Pvt.Ltd.and any such violation would entail initiation of suitable legal proceedings.

Copyright

Login ID:8FA03376/Student Name:ANSHULGARG/Overall Score:0

Question: 177
Read the passage below and answer the questions that follow.
What could be the moral of the above passage?
Every once in a while, a superhuman will come to save the earth from wicked people.

2)

The creation of the earth required a long time and great effort and human beings are not worthy of it.

3)

It is imperative that humans use the resources of the earth wisely and live in harmony amongst themselves otherwise
calamity could befall them.

4)

Fearless warriors always rescue people in trouble.

ite
d

1)

Li
m

Explanation:
Refer to the last two paragraphs. The monster Dragga is a metaphor for any calamitywhich could befall humans if they do not use the
resources of the earth wisely.Hence, [3].

iv
at
e

Question: 178
Read the passage below and answer the questions that follow.

good and evil.

2)

day and night.

3)

equal and complementary forces.

4)

precursors of man and woman.

es

1)

Pr

In the given passage, darkness and light symbolise:

ou

rc

Explanation:
Refer to paragraphs 1 and 5. Dark had more strength, while light was cunning but still when they fought they were equal implying that
each complemented the other. Hence, [3].

Why did Terra send Hercules to slay Dragga?

es

Question: 179
Read the passage below and answer the questions that follow.

He realized his folly in sending such a monster and that the future of the human race was at stake.

2)

When the humans cried out to Terra for help, he melted.

3)

He realized that Hercules could be a good king to the humans.

4)

As he had created the human race, he had a soft corner for humans.

ar

ni

ng

1)

Le

Explanation:
Refer to the last two paragraphs. Terra sent Dragga to teach human beings a lesson. But when they cried out for help, Terra had mercy
on them and he sent Hercules to slay Dragga. Hence, [2].

IM

Question: 180
Read the passage below and answer the questions that follow.
Life became possible on the universe due to the existence of:
1)

the Earth.

2)

the Sun.

3)

Thrakath and Tria.

4)

Terra.

IMS Learning Resources Pvt.Ltd.,Mumbai.All copyrights to this material vestswith IMS Learning Resources Pvt.Ltd.
No part of this materials either in part oras a whole shall be copied,printed,electronically reproduced,sold or distributed without the written
consent of IMS Learing Resources Pvt.Ltd.and any such violation would entail initiation of suitable legal proceedings.

Copyright

Login ID:8FA03376/Student Name:ANSHULGARG/Overall Score:0

Explanation:
Refer to paragraph 9, 3rdline the sun warmed the once cold universe so that life would be possible. Hence, [2].

bequeathal : legacy

2)

fortitude : mettle

3)

infirmity : fitness

4)

humility : subservience

Explanation:
The terms in the given pair are synonyms. But the terms in option [3]are antonyms. Hence, [3].

Li
m

1)

ite
d

Question: 181
Each question consists of two capitalized words that have a certain relationship to each other, followed by four pairs of words. Choose the
pair that is NOT RELATED mutually in the same way as the capitalized pair.
Acuity : Perspicacity

priest: hinduism

2)

imam: islam

3)

friar: christianity

4)

ayatollah : shinto

rc

es

Pr

1)

iv
at
e

Question: 182
Each question consists of two capitalized words that have a certain relationship to each other, followed by four pairs of words. Choose the
pair that is NOT RELATED mutually in the same way as the capitalized pair.
Rabbi : Judaism

ou

Explanation:
The relationship is that of a religious official and the religion he is associated with. 'Ayatollah' is associated with 'Islam'. Hence, [4].

peer : darkness

3)

eye : suspicion

4)

glance : passion

ng

2)

ni

stare : disbelief

ar

1)

es

Question: 183
Each question consists of two capitalized words that have a certain relationship to each other, followed by four pairs of words. Choose the
pair that is NOT RELATED mutually in the same way as the capitalized pair.
Glare : Anger

Le

Explanation:
You 'glare' in 'anger'. Similarly you 'stare' in 'disbelief', 'peer' in 'darkness', 'eye' in 'suspicion'. But you 'glance' quickly or cursorily, not with
'passion'. Hence, [4].

IM

Question: 184
Choose the option that is CLOSEST in meaning to the capitalized word.
KINDRED
1)

allied

2)

reciprocal

3)

different

4)

humane

IMS Learning Resources Pvt.Ltd.,Mumbai.All copyrights to this material vestswith IMS Learning Resources Pvt.Ltd.
No part of this materials either in part oras a whole shall be copied,printed,electronically reproduced,sold or distributed without the written
consent of IMS Learing Resources Pvt.Ltd.and any such violation would entail initiation of suitable legal proceedings.

Copyright

Login ID:8FA03376/Student Name:ANSHULGARG/Overall Score:0

Explanation:
'Kindred' means 'allied', connected or similar. Hence,[1].
Question: 185
Choose the option that is CLOSEST in meaning to the capitalized word.

pursuit

2)

obstruction

3)

industry

4)

maze

Li
m

1)

ite
d

LABYRINTH

Explanation:
'Labyrinth' means a complicated, irregular structure with many passages through which it is difficult to find one's way. Hence, [4].

2)

musical

3)

lengthy

4)

repetitive

Pr

bookish

es

1)

iv
at
e

Question: 186
Choose the option that is CLOSEST in meaning to the capitalized word.
PROLIX

ou

rc

Explanation:
'Prolix' is used with reference to a speech or writing that is quite 'lengthy'. Hence, [3].

es

Question: 187
Choose the option that is CLOSEST in meaning to the capitalized word.
INCORRIGIBLE
dependable

2)

principled

3)

reasonable

4)

uncorrectable

ni

ng

1)

ar

Explanation:
'Incorrigible' refers to someone who cannot be corrected or improved. Hence, [4].

creative

IM

1)

Le

Question: 188
Choose the option that is CLOSEST in meaning to the capitalized word.
PROMETHEAN

2)

enormous

3)

gigantic

4)

weary

Explanation:
'Promethean' means 'creative' or defiantly original. Hence, [1].
Question: 189

IMS Learning Resources Pvt.Ltd.,Mumbai.All copyrights to this material vestswith IMS Learning Resources Pvt.Ltd.
No part of this materials either in part oras a whole shall be copied,printed,electronically reproduced,sold or distributed without the written
consent of IMS Learing Resources Pvt.Ltd.and any such violation would entail initiation of suitable legal proceedings.

Copyright

Login ID:8FA03376/Student Name:ANSHULGARG/Overall Score:0

Choose the option that is CLOSEST in meaning to the capitalized word.


AMELIORATE
1)

indulge

2)

improve

3)

enable

4)

remove

ite
d

Explanation:
To 'ameliorate' means to make something better. Hence, [2].

double

2)

sickly

3)

drawn

4)

delicate

iv
at
e

1)

Li
m

Question: 190
Choose the option that is CLOSEST in meaning to the capitalized word.
DIAPHANOUS

2)

indispensable

3)

insolent

4)

susceptible

ou

voluntary

es

1)

rc

es

Question: 191
Choose the option that is OPPOSITE in meaning to the capitalized word.
Gratuitous

Pr

Explanation:
'Diaphanous' means very sheer or 'delicate'. The original Greek meaning is transparent. Hence, [4].

ng

Explanation:
The word 'gratuitous' means unnecessary or needless. Hence, [2].

ni

Question: 192
Choose the option that is OPPOSITE in meaning to the capitalized word.
APATHETIC
lost

2)

synthetic

3)

enthusiastic

4)

even-tempered

Le

ar

1)

IM

Explanation:
'Apathetic' is lacking interest or energy or being indifferent. Hence, [3].
Question: 193
Choose the option that is OPPOSITE in meaning to the capitalized word.
OPAQUE
1)

thick

2)

clear

IMS Learning Resources Pvt.Ltd.,Mumbai.All copyrights to this material vestswith IMS Learning Resources Pvt.Ltd.
No part of this materials either in part oras a whole shall be copied,printed,electronically reproduced,sold or distributed without the written
consent of IMS Learing Resources Pvt.Ltd.and any such violation would entail initiation of suitable legal proceedings.

Copyright

Login ID:8FA03376/Student Name:ANSHULGARG/Overall Score:0

3)

permeable

4)

turgid

Explanation:
'Opaque' means which is not transparent or which stops light from passing through it. If the substance is 'clear', it allows light to go through
it. Hence, [2].

diffident

2)

venal

3)

depraved

4)

recalcitrant

Li
m

1)

ite
d

Question: 194
Choose the option that is OPPOSITE in meaning to the capitalized word.
BRASH

2)

affable

3)

similar

4)

equal

Pr

ambiguous

ou

rc

1)

es

Question: 195
Choose the option that is OPPOSITE in meaning to the capitalized word.
UNEQUIVOCAL

iv
at
e

Explanation:
'Brash' means 'bold'. Hence, its opposite would be 'diffident'. Hence, [1].

es

Explanation:
'Unequivocal' means 'unambiguous'. Hence, [1].

IM

Le

ar

ni

ng

Question: 196
Read the passage below and answer the questions that follow.
Cloning is a much-maligned topic, even amongst the educated, the elite and the intelligentsia. Most of the opposition stems from
unfounded fear, misplaced moral and ethical indignation. Few people understand the dire medical need for cloning in a large number of
infertile couples. Cloning seems to be the only option now for them to achieve their biological children.
Cloning, it has been claimed, is still not a perfect technique. No medical technique was perfect when it was initiated. The technique was
improved upon over time. For cloning to improve, research work must go on with donated oocytes.
Cloning is now well established in veterinary practice. The number of species cloned is ever increasing. Ten species have so far been
cloned. Dolly was born after 277 oocytes were subjected to cloning. Since then, cloning efficiency has increased remarkably and today the
success rate is around 15 per cent. Seventy five percent of cloned animals are observed to be healthy. Cloned animals have mated
successfully and produced healthy litters.
This is the right moment to start research work on `human reproductive cloning'. It may take several years before a healthy human clone
can be produced. We must keep our minds open to reproductive cloning as one of the options for couples with otherwise incurable
infertility.
There are a large group of non-obstructive azoospermic men in whom no viable spermatozoa or haploid sperm precursors could be
obtained from their testes. These men can have a biological child only by cloning unless haploidisation (making a germ cell) becomes an
established technique. Similarly, for several women who have gone in for premature ovarian failure due to chromosomal or other causes,
the only hope to achieve motherhood would be by cloning unless haploidisation becomes a reality.
It has been argued that cloning produces carbon copies of individuals. Clones are no more identical than naturally occurring identical
twins; if anything, they are less identical. The mitochondrial genome of clones is different from the parent. Although the natural clones

IMS Learning Resources Pvt.Ltd.,Mumbai.All copyrights to this material vestswith IMS Learning Resources Pvt.Ltd.
No part of this materials either in part oras a whole shall be copied,printed,electronically reproduced,sold or distributed without the written
consent of IMS Learing Resources Pvt.Ltd.and any such violation would entail initiation of suitable legal proceedings.

Copyright

Login ID:8FA03376/Student Name:ANSHULGARG/Overall Score:0

iv
at
e

Li
m

ite
d

have identical genomes, they do not have identical phenotypes and behaviour. It has been well documented that cloned animals are
different from the parent, in both physical features and behaviour.
Extrapolating animal data may be insufficient to claim human cloning and will also be inefficient. There is a huge interspecies variation in
the success rate of several assisted reproductive technologies. There are indications to suggest that many of the anomalies seen in
animal work may not be present with human species. New research from Duke University Medical Centre suggests that cloning human
beings may not be as difficult as cloning livestock and other animals.
The major stumbling block to research on human reproductive cloning today is the scientists themselves. It is true that cloning is still not a
perfect technique. So were most of the medical techniques when they were introduced. Renal transplant was not a perfect technique
when it was introduced. Several patients died before the technique became successful. The inventors of the technique were labelled as
murderers though they went on to receive the Nobel Prize later on. Similarly, the first cardiac transplant patient lived for just 23 days!
Lesley Brown was told that she had a one in million chance of having a test tube baby. She pursued that one in a million chance and
today we have more than a million IVF babies in the world.
Would society accept a clone? The same issue was raised more than 26 years back, with IVF. Prof. Edwards was told that their work on
IVF was "unethical medical experimentation on possible future human beings and therefore it is subject to absolute moral prohibition".
Millions of babies have been born by ART and have been accepted by society. Regardless of the ethical, legal and moral issues, society
will accept things which are of common good.

The author's primary purpose in the above passage is:


to advocate the safety of the cloning process.

2)

to present a case on behalf of infertile couples.

3)

to present a case in defence of human reproductive cloning.

4)

to advocate the cause of continued research in cloning and its application to human reproductive concerns.

es

Pr

1)

es

ou

rc

Explanation:
The author, in the given passage, primarily tries to point out that it is the right moment for further research work on 'human reproductive
cloning' after the successes met with in the animal kingdom. In this context, he presents a case in defence of human reproductive cloning.
Hence, [4].

Question: 197
Why are the scientists themselves wary about cloning

They feel that they are likely to burn their own fingers in the process.

2)

They are not yet convinced about the safety and efficacy of cloning.

3)

They do not feel that infertile couples are a significant chunk of the population who could benefit from cloning

4)

They are not interested in research which does not directly concern them

ar

ni

ng

1)

Explanation:

Le

Solutions: Refer to paragraph 8. The major stumbling block to research on human reproductive cloning today is
scientists themselves as they feel that it is still not a perfect technique. This means that they are not yet
convinced about the safety and efficacy of cloning. Hence, [2].

IM

Question: 198
The author of the above passage is most likely to be:
1)

anandrologist

2)

ascientist

3)

aresearcher.

4)

cannot be determined.

Explanation:

IMS Learning Resources Pvt.Ltd.,Mumbai.All copyrights to this material vestswith IMS Learning Resources Pvt.Ltd.
No part of this materials either in part oras a whole shall be copied,printed,electronically reproduced,sold or distributed without the written
consent of IMS Learing Resources Pvt.Ltd.and any such violation would entail initiation of suitable legal proceedings.

Copyright

Login ID:8FA03376/Student Name:ANSHULGARG/Overall Score:0

On the basis of the information, we cannot surely deduce the profession of the author as he could be anyone from a scientist to a featurewriter. Hence, [4].

1)

Itoffers the best hope for a large section of childless couples.

2)

Most new procedures were imperfect to start with.

3)

Scientific development cannot be hampered by legislation by humans

4)

Society will always accept any procedure regardless of the human cost as long as it is beneficial.

ite
d

Question: 199
According to the author, why should we pursue cloning even though it is not a perfect technique?

Li
m

Explanation:
According to the author, we must keep our minds open to reproductive cloning as one of the options for couples with otherwise incurable
infertility. [4]cannotbe the answer as it makes a sweepinggeneralisation. Hence, [1].

iv
at
e

Question: 200
Inwhat way are clones less identical than twins?
Twins are created naturally and not in a laboratory.

2)

Twins have genes of both parents whereas clones are created from a single individual

3)

The mitochondrial genome of clones is different from the parent

4)

Clones can be created from anyone whereas not everyone can have twins

Pr

1)

IM

Le

ar

ni

ng

es

ou

rc

es

Explanation:
Refer to paragraph 6. The 1st three sentences clearly indicate the difference between natural twins and clones. Hence, [3]

IMS Learning Resources Pvt.Ltd.,Mumbai.All copyrights to this material vestswith IMS Learning Resources Pvt.Ltd.
No part of this materials either in part oras a whole shall be copied,printed,electronically reproduced,sold or distributed without the written
consent of IMS Learing Resources Pvt.Ltd.and any such violation would entail initiation of suitable legal proceedings.

Copyright

You might also like